Programs & Examples On #Street address

Street address refers to postal addresses, or delivery points, that can be used for physical delivery of mail or goods.

Address validation using Google Maps API

Validate it against FedEx's api. They have an API to generate labels from XML code. The process involves a step to validate the address.

Android: Reverse geocoding - getFromLocation

It looks like there's two things happening here.

1) You've missed the new keyword from before calling the constructor.

2) The parameter you're passing in to the Geocoder constructor is incorrect. You're passing in a Locale where it's expecting a Context.

There are two Geocoder constructors, both of which require a Context, and one also taking a Locale:

Geocoder(Context context, Locale locale)
Geocoder(Context context)

Solution

Modify your code to pass in a valid Context and include new and you should be good to go.

Geocoder myLocation = new Geocoder(getApplicationContext(), Locale.getDefault());   
List<Address> myList = myLocation.getFromLocation(latPoint, lngPoint, 1);

Note

If you're still having problems it may be a permissioning issue. Geocoding implicitly uses the Internet to perform the lookups, so your application will require an INTERNET uses-permission tag in your manifest.

Add the following uses-permission node within the manifest node of your manifest.

<uses-permission android:name="android.permission.INTERNET" />

mysql datatype for telephone number and address

Well, personally I do not use numeric datatype to store phone numbers or related info.

How do you store a number say 001234567? It'll end up as 1234567, losing the leading zeros.

Of course you can always left-pad it up, but that's provided you know exactly how many digits the number should be.

This doesn't answer your entire post,
Just my 2 cents

How to convert an address to a latitude/longitude?

You want a geocoding application. These are available either online or as an application backend.

How to parse freeform street/postal address out of text, and into components

I'm late to the party, here is an Excel VBA script I wrote years ago for Australia. It can be easily modified to support other Countries. I've made a GitHub repository of the C# code here. I've hosted it on my site and you can download it here: http://jeremythompson.net/rocks/ParseAddress.xlsm

Strategy

For any country with a PostCode that's numeric or can be matched with a RegEx my strategy works very well:

  1. First we detect the First and Surname which are assumed to be the top line. Its easy to skip the name and start with the address by unticking the checkbox (called 'Name is top row' as shown below).

  2. Next its safe to expect the Address consisting of the Street and Number come before the Suburb and the St, Pde, Ave, Av, Rd, Cres, loop, etc is a separator.

  3. Detecting the Suburb vs the State and even Country can trick the most sophisticated parsers as there can be conflicts. To overcome this I use a PostCode look up based on the fact that after stripping Street and Apartment/Unit numbers as well as the PoBox,Ph,Fax,Mobile etc, only the PostCode number will remain. This is easy to match with a regEx to then look up the suburb(s) and country.

Your National Post Office Service will provide a list of post codes with Suburbs and States free of charge that you can store in an excel sheet, db table, text/json/xml file, etc.

  1. Finally, since some Post Codes have multiple Suburbs we check which suburb appears in the Address.

Example

enter image description here

VBA Code

DISCLAIMER, I know this code is not perfect, or even written well however its very easy to convert to any programming language and run in any type of application.The strategy is the answer depending on your country and rules, take this code as an example:

Option Explicit

Private Const TopRow As Integer = 0

Public Sub ParseAddress()
Dim strArr() As String
Dim sigRow() As String
Dim i As Integer
Dim j As Integer
Dim k As Integer
Dim Stat As String
Dim SpaceInName As Integer
Dim Temp As String
Dim PhExt As String

On Error Resume Next

Temp = ActiveSheet.Range("Address")

'Split info into array
strArr = Split(Temp, vbLf)

'Trim the array
For i = 0 To UBound(strArr)
strArr(i) = VBA.Trim(strArr(i))
Next i

'Remove empty items/rows    
ReDim sigRow(LBound(strArr) To UBound(strArr))
For i = LBound(strArr) To UBound(strArr)
    If Trim(strArr(i)) <> "" Then
        sigRow(j) = strArr(i)
        j = j + 1
    End If
Next i
ReDim Preserve sigRow(LBound(strArr) To j)

'Find the name (MUST BE ON THE FIRST ROW UNLESS CHECKBOX UNTICKED)
i = TopRow
If ActiveSheet.Shapes("chkFirst").ControlFormat.Value = 1 Then

SpaceInName = InStr(1, sigRow(i), " ", vbTextCompare) - 1

If ActiveSheet.Shapes("chkConfirm").ControlFormat.Value = 0 Then
ActiveSheet.Range("FirstName") = VBA.Left(sigRow(i), SpaceInName)
Else
 If MsgBox("First Name: " & VBA.Mid$(sigRow(i), 1, SpaceInName), vbQuestion + vbYesNo, "Confirm Details") = vbYes Then ActiveSheet.Range("FirstName") = VBA.Left(sigRow(i), SpaceInName)
End If

If ActiveSheet.Shapes("chkConfirm").ControlFormat.Value = 0 Then
ActiveSheet.Range("Surname") = VBA.Mid(sigRow(i), SpaceInName + 2)
Else
  If MsgBox("Surame: " & VBA.Mid(sigRow(i), SpaceInName + 2), vbQuestion + vbYesNo, "Confirm Details") = vbYes Then ActiveSheet.Range("Surname") = VBA.Mid(sigRow(i), SpaceInName + 2)
End If
sigRow(i) = ""
End If

'Find the Street by looking for a "St, Pde, Ave, Av, Rd, Cres, loop, etc"
For i = 1 To UBound(sigRow)
If Len(sigRow(i)) > 0 Then
    For j = 0 To 8
    If InStr(1, VBA.UCase(sigRow(i)), Street(j), vbTextCompare) > 0 Then

    'Find the position of the street in order to get the suburb
    SpaceInName = InStr(1, VBA.UCase(sigRow(i)), Street(j), vbTextCompare) + Len(Street(j)) - 1

    'If its a po box then add 5 chars
    If VBA.Right(Street(j), 3) = "BOX" Then SpaceInName = SpaceInName + 5

    If ActiveSheet.Shapes("chkConfirm").ControlFormat.Value = 0 Then
    ActiveSheet.Range("Street") = VBA.Mid(sigRow(i), 1, SpaceInName)
    Else
      If MsgBox("Street Address: " & VBA.Mid(sigRow(i), 1, SpaceInName), vbQuestion + vbYesNo, "Confirm Details") = vbYes Then ActiveSheet.Range("Street") = VBA.Mid(sigRow(i), 1, SpaceInName)
    End If
    'Trim the Street, Number leaving the Suburb if its exists on the same line
    sigRow(i) = VBA.Mid(sigRow(i), SpaceInName) + 2
    sigRow(i) = Replace(sigRow(i), VBA.Mid(sigRow(i), 1, SpaceInName), "")

    GoTo PastAddress:
    End If
    Next j
End If
Next i
PastAddress:

'Mobile
For i = 1 To UBound(sigRow)
If Len(sigRow(i)) > 0 Then
    For j = 0 To 3
    Temp = Mb(j)
        If VBA.Left(VBA.UCase(sigRow(i)), Len(Temp)) = Temp Then
        If ActiveSheet.Shapes("chkConfirm").ControlFormat.Value = 0 Then
        ActiveSheet.Range("Mobile") = VBA.Mid(sigRow(i), Len(Temp) + 2)
        Else
          If MsgBox("Mobile: " & VBA.Mid(sigRow(i), Len(Temp) + 2), vbQuestion + vbYesNo, "Confirm Details") = vbYes Then ActiveSheet.Range("Mobile") = VBA.Mid(sigRow(i), Len(Temp) + 2)
        End If
    sigRow(i) = ""
    GoTo PastMobile:
    End If
    Next j
End If
Next i
PastMobile:

'Phone
For i = 1 To UBound(sigRow)
If Len(sigRow(i)) > 0 Then
    For j = 0 To 1
    Temp = Ph(j)
        If VBA.Left(VBA.UCase(sigRow(i)), Len(Temp)) = Temp Then

            'TODO: Detect the intl or national extension here.. or if we can from the postcode.
            If ActiveSheet.Shapes("chkConfirm").ControlFormat.Value = 0 Then
            ActiveSheet.Range("Phone") = VBA.Mid(sigRow(i), Len(Temp) + 3)
            Else
              If MsgBox("Phone: " & VBA.Mid(sigRow(i), Len(Temp) + 3), vbQuestion + vbYesNo, "Confirm Details") = vbYes Then ActiveSheet.Range("Phone") = VBA.Mid(sigRow(i), Len(Temp) + 3)
            End If

        sigRow(i) = ""
        GoTo PastPhone:
        End If
    Next j
End If
Next i
PastPhone:


'Email
For i = 1 To UBound(sigRow)
    If Len(sigRow(i)) > 0 Then
        'replace with regEx search
        If InStr(1, sigRow(i), "@", vbTextCompare) And InStr(1, VBA.UCase(sigRow(i)), ".CO", vbTextCompare) Then
        Dim email As String
        email = sigRow(i)
        email = Replace(VBA.UCase(email), "EMAIL:", "")
        email = Replace(VBA.UCase(email), "E-MAIL:", "")
        email = Replace(VBA.UCase(email), "E:", "")
        email = Replace(VBA.UCase(Trim(email)), "E ", "")
        email = VBA.LCase(email)

            If ActiveSheet.Shapes("chkConfirm").ControlFormat.Value = 0 Then
            ActiveSheet.Range("Email") = email
            Else
              If MsgBox("Email: " & email, vbQuestion + vbYesNo, "Confirm Details") = vbYes Then ActiveSheet.Range("Email") = email
            End If
        sigRow(i) = ""
        Exit For
        End If
    End If
Next i

'Now the only remaining items will be the postcode, suburb, country
'there shouldn't be any numbers (eg. from PoBox,Ph,Fax,Mobile) except for the Post Code

'Join the string and filter out the Post Code
Temp = Join(sigRow, vbCrLf)
Temp = Trim(Temp)

For i = 1 To Len(Temp)

Dim postCode As String
postCode = VBA.Mid(Temp, i, 4)

'In Australia PostCodes are 4 digits
If VBA.Mid(Temp, i, 1) <> " " And IsNumeric(postCode) Then

    If ActiveSheet.Shapes("chkConfirm").ControlFormat.Value = 0 Then
    ActiveSheet.Range("PostCode") = postCode
    Else
      If MsgBox("Post Code: " & postCode, vbQuestion + vbYesNo, "Confirm Details") = vbYes Then ActiveSheet.Range("PostCode") = postCode
    End If

    'Lookup the Suburb and State based on the PostCode, the PostCode sheet has the lookup
    Dim mySuburbArray As Range
    Set mySuburbArray = Sheets("PostCodes").Range("A2:B16670")

    Dim suburbs As String
    For j = 1 To mySuburbArray.Columns(1).Cells.Count
    If mySuburbArray.Cells(j, 1) = postCode Then
        'Check if the suburb is listed in the address
        If InStr(1, UCase(Temp), mySuburbArray.Cells(j, 2), vbTextCompare) > 0 Then

        'Set the Suburb and State
        ActiveSheet.Range("Suburb") = mySuburbArray.Cells(j, 2)
        Stat = mySuburbArray.Cells(j, 3)
        ActiveSheet.Range("State") = Stat

        'Knowing the State - for Australia we can get the telephone Ext
        PhExt = PhExtension(VBA.UCase(Stat))
        ActiveSheet.Range("PhExt") = PhExt

        'remove the phone extension from the number
        Dim prePhone As String
        prePhone = ActiveSheet.Range("Phone")
        prePhone = Replace(prePhone, PhExt & " ", "")
        prePhone = Replace(prePhone, "(" & PhExt & ") ", "")
        prePhone = Replace(prePhone, "(" & PhExt & ")", "")
        ActiveSheet.Range("Phone") = prePhone
        Exit For
        End If
    End If
    Next j
Exit For
End If
Next i

End Sub


Private Function PhExtension(ByVal State As String) As String
Select Case State
Case Is = "NSW"
PhExtension = "02"
Case Is = "QLD"
PhExtension = "07"
Case Is = "VIC"
PhExtension = "03"
Case Is = "NT"
PhExtension = "04"
Case Is = "WA"
PhExtension = "05"
Case Is = "SA"
PhExtension = "07"
Case Is = "TAS"
PhExtension = "06"
End Select
End Function

Private Function Ph(ByVal Num As Integer) As String
Select Case Num
Case Is = 0
Ph = "PH"
Case Is = 1
Ph = "PHONE"
'Case Is = 2
'Ph = "P"
End Select
End Function

Private Function Mb(ByVal Num As Integer) As String
Select Case Num
Case Is = 0
Mb = "MB"
Case Is = 1
Mb = "MOB"
Case Is = 2
Mb = "CELL"
Case Is = 3
Mb = "MOBILE"
'Case Is = 4
'Mb = "M"
End Select
End Function

Private Function Fax(ByVal Num As Integer) As String
Select Case Num
Case Is = 0
Fax = "FAX"
Case Is = 1
Fax = "FACSIMILE"
'Case Is = 2
'Fax = "F"
End Select
End Function

Private Function State(ByVal Num As Integer) As String
Select Case Num
Case Is = 0
State = "NSW"
Case Is = 1
State = "QLD"
Case Is = 2
State = "VIC"
Case Is = 3
State = "NT"
Case Is = 4
State = "WA"
Case Is = 5
State = "SA"
Case Is = 6
State = "TAS"
End Select
End Function

Private Function Street(ByVal Num As Integer) As String
Select Case Num
Case Is = 0
Street = " ST"
Case Is = 1
Street = " RD"
Case Is = 2
Street = " AVE"
Case Is = 3
Street = " AV"
Case Is = 4
Street = " CRES"
Case Is = 5
Street = " LOOP"
Case Is = 6
Street = "PO BOX"
Case Is = 7
Street = " STREET"
Case Is = 8
Street = " ROAD"
Case Is = 9
Street = " AVENUE"
Case Is = 10
Street = " CRESENT"
Case Is = 11
Street = " PARADE"
Case Is = 12
Street = " PDE"
Case Is = 13
Street = " LANE"
Case Is = 14
Street = " COURT"
Case Is = 15
Street = " BLVD"
Case Is = 16
Street = "P.O. BOX"
Case Is = 17
Street = "P.O BOX"
Case Is = 18
Street = "PO BOX"
Case Is = 19
Street = "POBOX"
End Select
End Function

Using Address Instead Of Longitude And Latitude With Google Maps API

See this example, initializes the map to "San Diego, CA".

Uses the Google Maps Javascript API v3 Geocoder to translate the address into coordinates that can be displayed on the map.

<html>
<head>
<meta name="viewport" content="initial-scale=1.0, user-scalable=no"/>
<meta http-equiv="content-type" content="text/html; charset=UTF-8"/>
<title>Google Maps JavaScript API v3 Example: Geocoding Simple</title>
<script type="text/javascript" src="http://maps.google.com/maps/api/js?sensor=false"></script>
<script type="text/javascript">
  var geocoder;
  var map;
  var address ="San Diego, CA";
  function initialize() {
    geocoder = new google.maps.Geocoder();
    var latlng = new google.maps.LatLng(-34.397, 150.644);
    var myOptions = {
      zoom: 8,
      center: latlng,
    mapTypeControl: true,
    mapTypeControlOptions: {style: google.maps.MapTypeControlStyle.DROPDOWN_MENU},
    navigationControl: true,
      mapTypeId: google.maps.MapTypeId.ROADMAP
    };
    map = new google.maps.Map(document.getElementById("map_canvas"), myOptions);
    if (geocoder) {
      geocoder.geocode( { 'address': address}, function(results, status) {
        if (status == google.maps.GeocoderStatus.OK) {
          if (status != google.maps.GeocoderStatus.ZERO_RESULTS) {
          map.setCenter(results[0].geometry.location);

            var infowindow = new google.maps.InfoWindow(
                { content: '<b>'+address+'</b>',
                  size: new google.maps.Size(150,50)
                });
    
            var marker = new google.maps.Marker({
                position: results[0].geometry.location,
                map: map, 
                title:address
            }); 
            google.maps.event.addListener(marker, 'click', function() {
                infowindow.open(map,marker);
            });

          } else {
            alert("No results found");
          }
        } else {
          alert("Geocode was not successful for the following reason: " + status);
        }
      });
    }
  }
</script>
</head>
<body style="margin:0px; padding:0px;" onload="initialize()">
 <div id="map_canvas" style="width:100%; height:100%">
</body>
</html>

working code snippet:

_x000D_
_x000D_
var geocoder;
var map;
var address = "San Diego, CA";

function initialize() {
  geocoder = new google.maps.Geocoder();
  var latlng = new google.maps.LatLng(-34.397, 150.644);
  var myOptions = {
    zoom: 8,
    center: latlng,
    mapTypeControl: true,
    mapTypeControlOptions: {
      style: google.maps.MapTypeControlStyle.DROPDOWN_MENU
    },
    navigationControl: true,
    mapTypeId: google.maps.MapTypeId.ROADMAP
  };
  map = new google.maps.Map(document.getElementById("map_canvas"), myOptions);
  if (geocoder) {
    geocoder.geocode({
      'address': address
    }, function(results, status) {
      if (status == google.maps.GeocoderStatus.OK) {
        if (status != google.maps.GeocoderStatus.ZERO_RESULTS) {
          map.setCenter(results[0].geometry.location);

          var infowindow = new google.maps.InfoWindow({
            content: '<b>' + address + '</b>',
            size: new google.maps.Size(150, 50)
          });

          var marker = new google.maps.Marker({
            position: results[0].geometry.location,
            map: map,
            title: address
          });
          google.maps.event.addListener(marker, 'click', function() {
            infowindow.open(map, marker);
          });

        } else {
          alert("No results found");
        }
      } else {
        alert("Geocode was not successful for the following reason: " + status);
      }
    });
  }
}
google.maps.event.addDomListener(window, 'load', initialize);
_x000D_
html,
body,
#map_canvas {
  height: 100%;
  width: 100%;
}
_x000D_
<script type="text/javascript" src="https://maps.google.com/maps/api/js?key=AIzaSyCkUOdZ5y7hMm0yrcCQoCvLwzdM6M8s5qk"></script>
<div id="map_canvas" ></div>
_x000D_
_x000D_
_x000D_

Parse usable Street Address, City, State, Zip from a string

RecogniContact is a Windows COM object that parses US and European addresses. You can try it right on http://www.loquisoft.com/index.php?page=8

How do you perform address validation?

I will refer you to my blog post - A lesson in address storage, I go into some of the techniques and algorithms used in the process of address validation. My key thought is "Don't be lazy with address storage, it will cause you nothing but headaches in the future!"

Also, there is another StackOverflow question that asks this question. Entitled How should international geographic addresses be stored in a relational database.

Setting up JUnit with IntelliJ IDEA

I needed to enable the JUnit plugin, after I linked my project with the jar files.

To enable the JUnit plugin, go to File->Settings, type "JUnit" in the search bar, and under "Plugins," check "JUnit.

vikingsteve's advice above will probably get the libraries linked right. Otherwise, open File->Project Structure, go to Libraries, hit the plus, and then browse to

C:\Program Files (x86)\JetBrains\IntelliJ IDEA Community Edition 14.1.1\lib\

and add these jar files:

hamcrest-core-1.3.jar
junit-4.11.jar 
junit.jar 

Getting file size in Python?

Try

os.path.getsize(filename)

It should return the size of a file, reported by os.stat().

String.equals versus ==

Let's analyze the following Java, to understand the identity and equality of Strings:

public static void testEquality(){
    String str1 = "Hello world.";
    String str2 = "Hello world.";

    if (str1 == str2)
        System.out.print("str1 == str2\n");
    else
        System.out.print("str1 != str2\n");

    if(str1.equals(str2))
        System.out.print("str1 equals to str2\n");
    else
        System.out.print("str1 doesn't equal to str2\n");

    String str3 = new String("Hello world.");
    String str4 = new String("Hello world.");

    if (str3 == str4)
        System.out.print("str3 == str4\n");
    else
        System.out.print("str3 != str4\n");

    if(str3.equals(str4))
        System.out.print("str3 equals to str4\n");
    else
        System.out.print("str3 doesn't equal to str4\n");
}

When the first line of code String str1 = "Hello world." executes, a string \Hello world." is created, and the variable str1 refers to it. Another string "Hello world." will not be created again when the next line of code executes because of optimization. The variable str2 also refers to the existing ""Hello world.".

The operator == checks identity of two objects (whether two variables refer to same object). Since str1 and str2 refer to same string in memory, they are identical to each other. The method equals checks equality of two objects (whether two objects have same content). Of course, the content of str1 and str2 are same.

When code String str3 = new String("Hello world.") executes, a new instance of string with content "Hello world." is created, and it is referred to by the variable str3. And then another instance of string with content "Hello world." is created again, and referred to by str4. Since str3 and str4 refer to two different instances, they are not identical, but their content are same.

Therefore, the output contains four lines:

Str1 == str2

Str1 equals str2

Str3! = str4

Str3 equals str4

Check if element at position [x] exists in the list

if(list.ElementAtOrDefault(2) != null)
{
   // logic
}

ElementAtOrDefault() is part of the System.Linq namespace.

Although you have a List, so you can use list.Count > 2.

ASP.NET MVC3 Razor - Html.ActionLink style

Reviving an old question because it seems to appear at the top of search results.

I wanted to retain transition effects while still being able to style the actionlink so I came up with this solution.

  1. I wrapped the action link with a div that would contain the parent style:
<div class="parent-style-one">
      @Html.ActionLink("Homepage", "Home", "Home")
</div>
  1. Next I create the CSS for the div, this will be the parent css and will be inherited by the child elements such as the action link.
  .parent-style-one {
     /* your styles here */
  }
  1. Because all an action link is, is an element when broken down as html so you just need to target that element in your css selection:
  .parent-style-one a {
     text-decoration: none;
  }
  1. For transition effects I did this:
  .parent-style-one a:hover {
        text-decoration: underline;
        -webkit-transition-duration: 1.1s; /* Safari */
        transition-duration: 1.1s;         
  }

This way I only target the child elements of the div in this case the action link and still be able to apply transition effects.

How to merge rows in a column into one cell in excel?

In simple cases you can use next method which doesn`t require you to create a function or to copy code to several cells:

  1. In any cell write next code

    =Transpose(A1:A9) 
    

Where A1:A9 are cells you would like to merge.

  1. Without leaving the cell press F9

After that, the cell will contain the string:

={A1,A2,A3,A4,A5,A6,A7,A8,A9}

Source: http://www.get-digital-help.com/2011/02/09/concatenate-a-cell-range-without-vba-in-excel/

Update: One part can be ambiguous. Without leaving the cell means having your cell in editor mode. Alternatevly you can press F9 while are in cell editor panel (normaly it can be found above the spreadsheet)

Dynamic function name in javascript?

You was near:

_x000D_
_x000D_
this["instance_" + a] = function () {...};
_x000D_
_x000D_
_x000D_

{...};

How to get the body's content of an iframe in Javascript?

The following code is cross-browser compliant. It works in IE7, IE8, Fx 3, Safari, and Chrome, so no need to handle cross-browser issues. Did not test in IE6.

<iframe id="iframeId" name="iframeId">...</iframe>

<script type="text/javascript">
    var iframeDoc;
    if (window.frames && window.frames.iframeId &&
        (iframeDoc = window.frames.iframeId.document)) {
        var iframeBody = iframeDoc.body;
        var ifromContent = iframeBody.innerHTML;
    }
</script>

Group by multiple field names in java 8

You have a few options here. The simplest is to chain your collectors:

Map<String, Map<Integer, List<Person>>> map = people
    .collect(Collectors.groupingBy(Person::getName,
        Collectors.groupingBy(Person::getAge));

Then to get a list of 18 year old people called Fred you would use:

map.get("Fred").get(18);

A second option is to define a class that represents the grouping. This can be inside Person. This code uses a record but it could just as easily be a class (with equals and hashCode defined) in versions of Java before JEP 359 was added:

class Person {
    record NameAge(String name, int age) { }

    public NameAge getNameAge() {
        return new NameAge(name, age);
    }
}

Then you can use:

Map<NameAge, List<Person>> map = people.collect(Collectors.groupingBy(Person::getNameAge));

and search with

map.get(new NameAge("Fred", 18));

Finally if you don't want to implement your own group record then many of the Java frameworks around have a pair class designed for this type of thing. For example: apache commons pair If you use one of these libraries then you can make the key to the map a pair of the name and age:

Map<Pair<String, Integer>, List<Person>> map =
    people.collect(Collectors.groupingBy(p -> Pair.of(p.getName(), p.getAge())));

and retrieve with:

map.get(Pair.of("Fred", 18));

Personally I don't really see much value in generic tuples now that records are available in the language as records display intent better and require very little code.

How to implode array with key and value without foreach in PHP

and another way:

$input = array(
    'item1'  => 'object1',
    'item2'  => 'object2',
    'item-n' => 'object-n'
);

$output = implode(', ', array_map(
    function ($v, $k) {
        if(is_array($v)){
            return $k.'[]='.implode('&'.$k.'[]=', $v);
        }else{
            return $k.'='.$v;
        }
    }, 
    $input, 
    array_keys($input)
));

or:

$output = implode(', ', array_map(
    function ($v, $k) { return sprintf("%s='%s'", $k, $v); },
    $input,
    array_keys($input)
));

read file in classpath

import java.io.BufferedReader;
import java.io.File;
import java.io.FileNotFoundException;
import java.io.FileReader;
import java.io.IOException;

public class readFile {
    /**
     * feel free to make any modification I have have been here so I feel you
     * 
     * @param args
     * @throws InterruptedException
     */
    public static void main(String[] args) throws InterruptedException {
        File dir = new File(".");// read file from same directory as source //
        if (dir.isDirectory()) {
            File[] files = dir.listFiles();
            for (File file : files) {
                // if you wanna read file name with txt files
                if (file.getName().contains("txt")) {
                    System.out.println(file.getName());
                }

                // if you want to open text file and read each line then
                if (file.getName().contains("txt")) {
                    try {
                        // FileReader reads text files in the default encoding.
                        FileReader fileReader = new FileReader(
                                file.getAbsolutePath());
                        // Always wrap FileReader in BufferedReader.
                        BufferedReader bufferedReader = new BufferedReader(
                                fileReader);
                        String line;
                        // get file details and get info you need.
                        while ((line = bufferedReader.readLine()) != null) {
                            System.out.println(line);
                            // here you can say...
                            // System.out.println(line.substring(0, 10)); this
                            // prints from 0 to 10 indext
                        }
                    } catch (FileNotFoundException ex) {
                        System.out.println("Unable to open file '"
                                + file.getName() + "'");
                    } catch (IOException ex) {
                        System.out.println("Error reading file '"
                                + file.getName() + "'");
                        // Or we could just do this:
                        ex.printStackTrace();
                    }
                }
            }
        }

    }`enter code here`

}

How to confirm RedHat Enterprise Linux version?

Avoid /etc/*release* files and run this command instead, it is far more reliable and gives more details:

rpm -qia '*release*'

How to convert Javascript datetime to C# datetime?

You could use the toJSON() JavaScript method, it converts a JavaScript DateTime to what C# can recognise as a DateTime.

The JavaScript code looks like this

var date = new Date();
date.toJSON(); // this is the JavaScript date as a c# DateTime

Note: The result will be in UTC time

Hash Map in Python

Here is the implementation of the Hash Map using python For the simplicity hash map is of a fixed size 16. This can be changed easily. Rehashing is out of scope of this code.

class Node:
    def __init__(self, key, value):
        self.key = key
        self.value = value
        self.next = None

class HashMap:
    def __init__(self):
        self.store = [None for _ in range(16)]
    def get(self, key):
        index = hash(key) & 15
        if self.store[index] is None:
            return None
        n = self.store[index]
        while True:
            if n.key == key:
                return n.value
            else:
                if n.next:
                    n = n.next
                else:
                    return None
    def put(self, key, value):
        nd = Node(key, value)
        index = hash(key) & 15
        n = self.store[index]
        if n is None:
            self.store[index] = nd
        else:
            if n.key == key:
                n.value = value
            else:
                while n.next:
                    if n.key == key:
                        n.value = value
                        return
                    else:
                        n = n.next
                n.next = nd

hm = HashMap()
hm.put("1", "sachin")
hm.put("2", "sehwag")
hm.put("3", "ganguly")
hm.put("4", "srinath")
hm.put("5", "kumble")
hm.put("6", "dhoni")
hm.put("7", "kohli")
hm.put("8", "pandya")
hm.put("9", "rohit")
hm.put("10", "dhawan")
hm.put("11", "shastri")
hm.put("12", "manjarekar")
hm.put("13", "gupta")
hm.put("14", "agarkar")
hm.put("15", "nehra")
hm.put("16", "gawaskar")
hm.put("17", "vengsarkar")
print(hm.get("1"))
print(hm.get("2"))
print(hm.get("3"))
print(hm.get("4"))
print(hm.get("5"))
print(hm.get("6"))
print(hm.get("7"))
print(hm.get("8"))
print(hm.get("9"))
print(hm.get("10"))
print(hm.get("11"))
print(hm.get("12"))
print(hm.get("13"))
print(hm.get("14"))
print(hm.get("15"))
print(hm.get("16"))
print(hm.get("17"))

Output:

sachin
sehwag
ganguly
srinath
kumble
dhoni
kohli
pandya
rohit
dhawan
shastri
manjarekar
gupta
agarkar
nehra
gawaskar
vengsarkar

How do you clear the console screen in C?

For portability, try this:

#ifdef _WIN32
#include <conio.h>
#else
#include <stdio.h>
#define clrscr() printf("\e[1;1H\e[2J")
#endif

Then simply call clrscr(). On Windows, it will use conio.h's clrscr(), and on Linux, it will use ANSI escape codes.

If you really want to do it "properly", you can eliminate the middlemen (conio, printf, etc.) and do it with just the low-level system tools (prepare for a massive code-dump):

#ifdef _WIN32
#define WIN32_LEAN_AND_MEAN
#include <windows.h>

void ClearScreen()
{
  HANDLE                     hStdOut;
  CONSOLE_SCREEN_BUFFER_INFO csbi;
  DWORD                      count;
  DWORD                      cellCount;
  COORD                      homeCoords = { 0, 0 };

  hStdOut = GetStdHandle( STD_OUTPUT_HANDLE );
  if (hStdOut == INVALID_HANDLE_VALUE) return;

  /* Get the number of cells in the current buffer */
  if (!GetConsoleScreenBufferInfo( hStdOut, &csbi )) return;
  cellCount = csbi.dwSize.X *csbi.dwSize.Y;

  /* Fill the entire buffer with spaces */
  if (!FillConsoleOutputCharacter(
    hStdOut,
    (TCHAR) ' ',
    cellCount,
    homeCoords,
    &count
    )) return;

  /* Fill the entire buffer with the current colors and attributes */
  if (!FillConsoleOutputAttribute(
    hStdOut,
    csbi.wAttributes,
    cellCount,
    homeCoords,
    &count
    )) return;

  /* Move the cursor home */
  SetConsoleCursorPosition( hStdOut, homeCoords );
}

#else // !_WIN32
#include <unistd.h>
#include <term.h>

void ClearScreen()
{
  if (!cur_term)
  {
     int result;
     setupterm( NULL, STDOUT_FILENO, &result );
     if (result <= 0) return;
  }

   putp( tigetstr( "clear" ) );
}
#endif

Determining Referer in PHP

What I have found best is a CSRF token and save it in the session for links where you need to verify the referrer.

So if you are generating a FB callback then it would look something like this:

$token = uniqid(mt_rand(), TRUE);
$_SESSION['token'] = $token;
$url = "http://example.com/index.php?token={$token}";

Then the index.php will look like this:

if(empty($_GET['token']) || $_GET['token'] !== $_SESSION['token'])
{
    show_404();
} 

//Continue with the rest of code

I do know of secure sites that do the equivalent of this for all their secure pages.

Rotate label text in seaborn factorplot

For a seaborn.heatmap, you can rotate these using (based on @Aman's answer)

pandas_frame = pd.DataFrame(data, index=names, columns=names)
heatmap = seaborn.heatmap(pandas_frame)
loc, labels = plt.xticks()
heatmap.set_xticklabels(labels, rotation=45)
heatmap.set_yticklabels(labels[::-1], rotation=45) # reversed order for y

How do you take a git diff file, and apply it to a local branch that is a copy of the same repository?

It seems like you can also use the patch command. Put the diff in the root of the repository and run patch from the command line.

patch -i yourcoworkers.diff

or

patch -p0 -i yourcoworkers.diff

You may need to remove the leading folder structure if they created the diff without using --no-prefix.

If so, then you can remove the parts of the folder that don't apply using:

patch -p1 -i yourcoworkers.diff

The -p(n) signifies how many parts of the folder structure to remove.

More information on creating and applying patches here.

You can also use

git apply yourcoworkers.diff --stat 

to see if the diff by default will apply any changes. It may say 0 files affected if the patch is not applied correctly (different folder structure).

insert datetime value in sql database with c#

This is an older question with a proper answer (please use parameterized queries) which I'd like to extend with some timezone discussion. For my current project I was interested in how do the datetime columns handle timezones and this question is the one I found.

Turns out, they do not, at all.

datetime column stores the given DateTime as is, without any conversion. It does not matter if the given datetime is UTC or local.

You can see for yourself:

using (var connection = new SqlConnection(connectionString))
{
    connection.Open();
    using (var command = connection.CreateCommand())
    {
        command.CommandText = "SELECT * FROM (VALUES (@a, @b, @c)) example(a, b, c);";

        var local = DateTime.Now;
        var utc = local.ToUniversalTime();

        command.Parameters.AddWithValue("@a", utc);
        command.Parameters.AddWithValue("@b", local);
        command.Parameters.AddWithValue("@c", utc.ToLocalTime());

        using (var reader = command.ExecuteReader())
        {
            reader.Read();

            var localRendered = local.ToString("o");

            Console.WriteLine($"a = {utc.ToString("o").PadRight(localRendered.Length, ' ')} read = {reader.GetDateTime(0):o}, {reader.GetDateTime(0).Kind}");
            Console.WriteLine($"b = {local:o} read = {reader.GetDateTime(1):o}, {reader.GetDateTime(1).Kind}");
            Console.WriteLine($"{"".PadRight(localRendered.Length + 4, ' ')} read = {reader.GetDateTime(2):o}, {reader.GetDateTime(2).Kind}");
        }
    }
}

What this will print will of course depend on your time zone but most importantly the read values will all have Kind = Unspecified. The first and second output line will be different by your timezone offset. Second and third will be the same. Using the "o" format string (roundtrip) will not show any timezone specifiers for the read values.

Example output from GMT+02:00:

a = 2018-11-20T10:17:56.8710881Z      read = 2018-11-20T10:17:56.8700000, Unspecified
b = 2018-11-20T12:17:56.8710881+02:00 read = 2018-11-20T12:17:56.8700000, Unspecified
                                      read = 2018-11-20T12:17:56.8700000, Unspecified

Also note of how the data gets truncated (or rounded) to what seems like 10ms.

Regular expression to limit number of characters to 10

It might be beneficial to add greedy matching to the end of the string, so you can accept strings > than 10 and the regex will only return up to the first 10 chars. /^[a-z0-9]{0,10}$?/

How to Bootstrap navbar static to fixed on scroll?

/** * Scroll management */ $(document).ready(function () {

// Define the menu we are working with
var menu = $('.navbar.navbar-default.navbar-inverse');

// Get the menus current offset
var origOffsetY = menu.offset().top;

/**
 * scroll
 * Perform our menu mod
 */
function scroll() {

    // Check the menus offset. 
    if ($(window).scrollTop() >= origOffsetY) {

        //If it is indeed beyond the offset, affix it to the top.
        $(menu).addClass('navbar-fixed-top');

    } else {

        // Otherwise, un affix it.
        $(menu).removeClass('navbar-fixed-top');

    }
}

// Anytime the document is scrolled act on it
document.onscroll = scroll;

});

The absolute uri: http://java.sun.com/jsp/jstl/core cannot be resolved in either web.xml or the jar files deployed with this application

Remove the standard.jar. It's apparently of old JSTL 1.0 version when the TLD URIs were without the /jsp path. With JSTL 1.2 as available here you don't need a standard.jar at all. Just the jstl-1.2.jar in /WEB-INF/lib is sufficient.

See also:

How can I find the maximum value and its index in array in MATLAB?

In case of a 2D array (matrix), you can use:

[val, idx] = max(A, [], 2);

The idx part will contain the column number of containing the max element of each row.

MySQL and PHP - insert NULL rather than empty string

This works just fine for me:

INSERT INTO table VALUES ('', NULLIF('$date',''))

(first '' increments id field)

How to change the button text of <input type="file" />?

You can simply add some css trick. you don't need javascript or more input files and i keep existing value attribute. you need to add only css. you can try this solution.

_x000D_
_x000D_
.btn-file-upload{_x000D_
     width: 187px;_x000D_
     position:relative;_x000D_
 }_x000D_
_x000D_
.btn-file-upload:after{_x000D_
    content:  attr(value);_x000D_
    position: absolute;_x000D_
    top: 0;_x000D_
    left: 0;_x000D_
    bottom: 0;    _x000D_
    width: 48%;_x000D_
    background: #795548;_x000D_
    color: white;_x000D_
    border-radius: 2px;_x000D_
    text-align: center;_x000D_
    font-size: 12px;_x000D_
    line-height: 2;_x000D_
 }
_x000D_
<input type="file" class="btn-file-upload" value="Uploadfile" />
_x000D_
_x000D_
_x000D_

Best approach to converting Boolean object to string in java

If you see implementation of both the method, they look same.

String.valueOf(b)

public static String valueOf(boolean b) {
        return b ? "true" : "false";
    }

Boolean.toString(b)

public static String toString(boolean b) {
        return b ? "true" : "false";
    }

So both the methods are equally efficient.

T-SQL Cast versus Convert

You should also not use CAST for getting the text of a hash algorithm. CAST(HASHBYTES('...') AS VARCHAR(32)) is not the same as CONVERT(VARCHAR(32), HASHBYTES('...'), 2). Without the last parameter, the result would be the same, but not a readable text. As far as I know, You cannot specify that last parameter in CAST.

Is it possible to display inline images from html in an Android TextView?

If you have a look at the documentation for Html.fromHtml(text) you'll see it says:

Any <img> tags in the HTML will display as a generic replacement image which your program can then go through and replace with real images.

If you don't want to do this replacement yourself you can use the other Html.fromHtml() method which takes an Html.TagHandler and an Html.ImageGetter as arguments as well as the text to parse.

In your case you could parse null as for the Html.TagHandler but you'd need to implement your own Html.ImageGetter as there isn't a default implementation.

However, the problem you're going to have is that the Html.ImageGetter needs to run synchronously and if you're downloading images from the web you'll probably want to do that asynchronously. If you can add any images you want to display as resources in your application the your ImageGetter implementation becomes a lot simpler. You could get away with something like:

private class ImageGetter implements Html.ImageGetter {

    public Drawable getDrawable(String source) {
        int id;

        if (source.equals("stack.jpg")) {
            id = R.drawable.stack;
        }
        else if (source.equals("overflow.jpg")) {
            id = R.drawable.overflow;
        }
        else {
            return null;
        }

        Drawable d = getResources().getDrawable(id);
        d.setBounds(0,0,d.getIntrinsicWidth(),d.getIntrinsicHeight());
        return d;
    }
};

You'd probably want to figure out something smarter for mapping source strings to resource IDs though.

Excel formula to reference 'CELL TO THE LEFT'

Please select the entire sheet and HOME > Styles - Conditional Formatting, New Rule..., Use a formula to determine which cells to format and Format values where this formula is true::

=A1<>XFD1

Format..., select choice of formatting, OK, OK.

C#: Waiting for all threads to complete

This may not be an option for you, but if you can use the Parallel Extension for .NET then you could use Tasks instead of raw threads and then use Task.WaitAll() to wait for them to complete.

PHP exec() vs system() vs passthru()

They have slightly different purposes.

  • exec() is for calling a system command, and perhaps dealing with the output yourself.
  • system() is for executing a system command and immediately displaying the output - presumably text.
  • passthru() is for executing a system command which you wish the raw return from - presumably something binary.

Regardless, I suggest you not use any of them. They all produce highly unportable code.

One time page refresh after first page load

After </body> tag:

<script type="text/javascript">
if (location.href.indexOf('reload')==-1)
{
   location.href=location.href+'?reload';
}
</script>

How to get all child inputs of a div element (jQuery)

You need

var i = $("#panel input"); 

or, depending on what exactly you want (see below)

var i = $("#panel :input"); 

the > will restrict to children, you want all descendants.

EDIT: As Nick pointed out, there's a subtle difference between $("#panel input") and $("#panel :input).

The first one will only retrieve elements of type input, that is <input type="...">, but not <textarea>, <button> and <select> elements. Thanks Nick, didn't know this myself and corrected my post accordingly. Left both options, because I guess the OP wasn't aware of that either and -technically- asked for inputs... :-)

Close Bootstrap modal on form submit

Use that Code

 $('#button').submit(function(e) {
    e.preventDefault();
    // Coding
    $('#IDModal').modal('toggle'); //or  $('#IDModal').modal('hide');
    return false;
});

How to draw a checkmark / tick using CSS?

This is simple css for Sign Mark.

ul li:after{opacity: 1;content: '\2713';right: 20px;position: absolute;font-size: 20px;font-weight: bold;}

Draw a line in a div

Answered this just to emphasize @rblarsen comment on question :

You don't need the style tags in the CSS-file

If you remove the style tag from your css file it will work.

How to get elements with multiple classes

actually @bazzlebrush 's answer and @filoxo 's comment helped me a lot.

I needed to find the elements where the class could be "zA yO" OR "zA zE"

Using jquery I first select the parent of the desired elements:

(a div with class starting with 'abc' and style != 'display:none')

var tom = $('div[class^="abc"][style!="display: none;"]')[0];                   

then the desired children of that element:

var ax = tom.querySelectorAll('.zA.yO, .zA.zE');

works perfectly! note you don't have to do document.querySelector you can as above pass in a pre-selected object.

Foreign Key to multiple tables

You have a few options, all varying in "correctness" and ease of use. As always, the right design depends on your needs.

  • You could simply create two columns in Ticket, OwnedByUserId and OwnedByGroupId, and have nullable Foreign Keys to each table.

  • You could create M:M reference tables enabling both ticket:user and ticket:group relationships. Perhaps in future you will want to allow a single ticket to be owned by multiple users or groups? This design does not enforce that a ticket must be owned by a single entity only.

  • You could create a default group for every user and have tickets simply owned by either a true Group or a User's default Group.

  • Or (my choice) model an entity that acts as a base for both Users and Groups, and have tickets owned by that entity.

Heres a rough example using your posted schema:

create table dbo.PartyType
(   
    PartyTypeId tinyint primary key,
    PartyTypeName varchar(10)
)

insert into dbo.PartyType
    values(1, 'User'), (2, 'Group');


create table dbo.Party
(
    PartyId int identity(1,1) primary key,
    PartyTypeId tinyint references dbo.PartyType(PartyTypeId),
    unique (PartyId, PartyTypeId)
)

CREATE TABLE dbo.[Group]
(
    ID int primary key,
    Name varchar(50) NOT NULL,
    PartyTypeId as cast(2 as tinyint) persisted,
    foreign key (ID, PartyTypeId) references Party(PartyId, PartyTypeID)
)  

CREATE TABLE dbo.[User]
(
    ID int primary key,
    Name varchar(50) NOT NULL,
    PartyTypeId as cast(1 as tinyint) persisted,
    foreign key (ID, PartyTypeId) references Party(PartyID, PartyTypeID)
)

CREATE TABLE dbo.Ticket
(
    ID int primary key,
    [Owner] int NOT NULL references dbo.Party(PartyId),
    [Subject] varchar(50) NULL
)

Sending HTTP POST Request In Java

The first answer was great, but I had to add try/catch to avoid Java compiler errors.
Also, I had troubles to figure how to read the HttpResponse with Java libraries.

Here is the more complete code :

/*
 * Create the POST request
 */
HttpClient httpClient = new DefaultHttpClient();
HttpPost httpPost = new HttpPost("http://example.com/");
// Request parameters and other properties.
List<NameValuePair> params = new ArrayList<NameValuePair>();
params.add(new BasicNameValuePair("user", "Bob"));
try {
    httpPost.setEntity(new UrlEncodedFormEntity(params, "UTF-8"));
} catch (UnsupportedEncodingException e) {
    // writing error to Log
    e.printStackTrace();
}
/*
 * Execute the HTTP Request
 */
try {
    HttpResponse response = httpClient.execute(httpPost);
    HttpEntity respEntity = response.getEntity();

    if (respEntity != null) {
        // EntityUtils to get the response content
        String content =  EntityUtils.toString(respEntity);
    }
} catch (ClientProtocolException e) {
    // writing exception to log
    e.printStackTrace();
} catch (IOException e) {
    // writing exception to log
    e.printStackTrace();
}

How to select clear table contents without destroying the table?

There is a condition that most of these solutions do not address. I revised Patrick Honorez's solution to handle it. I felt I had to share this because I was pulling my hair out when the original function was occasionally clearing more data that I expected.

The situation happens when the table only has one column and the .SpecialCells(xlCellTypeConstants).ClearContents attempts to clear the contents of the top row. In this situation, only one cell is selected (the top row of the table that only has one column) and the SpecialCells command applies to the entire sheet instead of the selected range. What was happening to me was other cells on the sheet that were outside of my table were also getting cleared.

I did some digging and found this advice from Mathieu Guindon: Range SpecialCells ClearContents clears whole sheet

Range({any single cell}).SpecialCells({whatever}) seems to work off the entire sheet.

Range({more than one cell}).SpecialCells({whatever}) seems to work off the specified cells.

If the list/table only has one column (in row 1), this revision will check to see if the cell has a formula and if not, it will only clear the contents of that one cell.

Public Sub ClearList(lst As ListObject)
'Clears a listObject while leaving 1 empty row + formula
' https://stackoverflow.com/a/53856079/1898524
'
'With special help from this post to handle a single column table.
'   Range({any single cell}).SpecialCells({whatever}) seems to work off the entire sheet.
'   Range({more than one cell}).SpecialCells({whatever}) seems to work off the specified cells.
' https://stackoverflow.com/questions/40537537/range-specialcells-clearcontents-clears-whole-sheet-instead

    On Error Resume Next
    
    With lst
        '.Range.Worksheet.Activate ' Enable this if you are debugging 
    
        If .ShowAutoFilter Then .AutoFilter.ShowAllData
        If .DataBodyRange.Rows.Count = 1 Then Exit Sub ' Table is already clear
        .DataBodyRange.Offset(1).Rows.Clear
        
        If .DataBodyRange.Columns.Count > 1 Then ' Check to see if SpecialCells is going to evaluate just one cell.
            .DataBodyRange.Rows(1).SpecialCells(xlCellTypeConstants).ClearContents
        ElseIf Not .Range.HasFormula Then
            ' Only one cell in range and it does not contain a formula.
            .DataBodyRange.Rows(1).ClearContents
        End If

        .Resize .Range.Rows("1:2")
        
        .HeaderRowRange.Offset(1).Select

        ' Reset used range on the sheet
        Dim X
        X = .Range.Worksheet.UsedRange.Rows.Count 'see J-Walkenbach tip 73

    End With

End Sub

A final step I included is a tip that is attributed to John Walkenbach, sometimes noted as J-Walkenbach tip 73 Automatically Resetting The Last Cell

How do you post to the wall on a facebook page (not profile)

You can make api calls by choosing the HTTP method and setting optional parameters:

$facebook->api('/me/feed/', 'post', array(
    'message' => 'I want to display this message on my wall'
));

Submit Post to Facebook Wall :

Include the fbConfig.php file to connect Facebook API and get the access token.

Post message, name, link, description, and the picture will be submitted to Facebook wall. Post submission status will be shown.

If FB access token ($accessToken) is not available, the Facebook Login URL will be generated and the user would be redirected to the FB login page.

Post to facebook wall php sdk

<?php
//Include FB config file
require_once 'fbConfig.php';

if(isset($accessToken)){
    if(isset($_SESSION['facebook_access_token'])){
        $fb->setDefaultAccessToken($_SESSION['facebook_access_token']);
    }else{
        // Put short-lived access token in session
        $_SESSION['facebook_access_token'] = (string) $accessToken;

        // OAuth 2.0 client handler helps to manage access tokens
        $oAuth2Client = $fb->getOAuth2Client();

        // Exchanges a short-lived access token for a long-lived one
        $longLivedAccessToken = $oAuth2Client->getLongLivedAccessToken($_SESSION['facebook_access_token']);
        $_SESSION['facebook_access_token'] = (string) $longLivedAccessToken;

        // Set default access token to be used in script
        $fb->setDefaultAccessToken($_SESSION['facebook_access_token']);
    }

    //FB post content
    $message = 'Test message from CodexWorld.com website';
    $title = 'Post From Website';
    $link = 'http://www.codexworld.com/';
    $description = 'CodexWorld is a programming blog.';
    $picture = 'http://www.codexworld.com/wp-content/uploads/2015/12/www-codexworld-com-programming-blog.png';

    $attachment = array(
        'message' => $message,
        'name' => $title,
        'link' => $link,
        'description' => $description,
        'picture'=>$picture,
    );

    try{
        //Post to Facebook
        $fb->post('/me/feed', $attachment, $accessToken);

        //Display post submission status
        echo 'The post was submitted successfully to Facebook timeline.';
    }catch(FacebookResponseException $e){
        echo 'Graph returned an error: ' . $e->getMessage();
        exit;
    }catch(FacebookSDKException $e){
        echo 'Facebook SDK returned an error: ' . $e->getMessage();
        exit;
    }
}else{
    //Get FB login URL
    $fbLoginURL = $helper->getLoginUrl($redirectURL, $fbPermissions);

    //Redirect to FB login
    header("Location:".$fbLoginURL);
}

Refrences:

https://github.com/facebookarchive/facebook-php-sdk

https://developers.facebook.com/docs/pages/publishing/

https://developers.facebook.com/docs/php/gettingstarted

http://www.pontikis.net/blog/auto_post_on_facebook_with_php

https://www.codexworld.com/post-to-facebook-wall-from-website-php-sdk/

Using TortoiseSVN via the command line

TortoiseSVN has a command-line interface that can be used for TortoiseSVN GUI automation and it's different from the normal Subversion one.

You can find information about the command-line options of TortoiseSVN in the documentation: Appendix D. Automating TortoiseSVN. The main program to work with here is TortoiseProc.exe.

But a note pretty much at the top there already says:

Remember that TortoiseSVN is a GUI client, and this automation guide shows you how to make the TortoiseSVN dialogs appear to collect user input. If you want to write a script which requires no input, you should use the official Subversion command line client instead.

Another option would be that you install the Subversion binaries. Slik SVN is a nice build (and doesn't require a registration like Collabnet). Recent versions of TortoiseSVN also include the command-line client if you choose to install it.

Failed to load resource: the server responded with a status of 404 (Not Found)

Your files are not under the jsp folder that's why it is not found. You have to go back again 1 folder Try this:

     <script  src="../../Jquery/prettify.js"></script>

How do I find out which settings.xml file maven is using

Use the Maven debug option, ie mvn -X :

Apache Maven 3.0.3 (r1075438; 2011-02-28 18:31:09+0100)
Maven home: /usr/java/apache-maven-3.0.3
Java version: 1.6.0_12, vendor: Sun Microsystems Inc.
Java home: /usr/java/jdk1.6.0_12/jre
Default locale: en_US, platform encoding: UTF-8
OS name: "linux", version: "2.6.32-32-generic", arch: "i386", family: "unix"
[INFO] Error stacktraces are turned on.
[DEBUG] Reading global settings from /usr/java/apache-maven-3.0.3/conf/settings.xml
[DEBUG] Reading user settings from /home/myhome/.m2/settings.xml
...

In this output, you can see that the settings.xml is loaded from /home/myhome/.m2/settings.xml.

Pandas split DataFrame by column value

Using groupby you could split into two dataframes like

In [1047]: df1, df2 = [x for _, x in df.groupby(df['Sales'] < 30)]

In [1048]: df1
Out[1048]:
   A  Sales
2  7     30
3  6     40
4  1     50

In [1049]: df2
Out[1049]:
   A  Sales
0  3     10
1  4     20

How to "git clone" including submodules?

You can use the --recursive flag when cloning a repository. This parameter forces git to clone all defined submodules in the repository.

git clone --recursive [email protected]:your_repo.git

After cloning, sometimes submodules branches may be changed, so run this command after it:

git submodule foreach "git checkout master"

Convert double to BigDecimal and set BigDecimal Precision

You want to try String.format("%f", d), which will print your double in decimal notation. Don't use BigDecimal at all.

Regarding the precision issue: You are first storing 47.48 in the double c, then making a new BigDecimal from that double. The loss of precision is in assigning to c. You could do

BigDecimal b = new BigDecimal("47.48")

to avoid losing any precision.

Catch multiple exceptions in one line (except block)

How do I catch multiple exceptions in one line (except block)

Do this:

try:
    may_raise_specific_errors():
except (SpecificErrorOne, SpecificErrorTwo) as error:
    handle(error) # might log or have some other default behavior...

The parentheses are required due to older syntax that used the commas to assign the error object to a name. The as keyword is used for the assignment. You can use any name for the error object, I prefer error personally.

Best Practice

To do this in a manner currently and forward compatible with Python, you need to separate the Exceptions with commas and wrap them with parentheses to differentiate from earlier syntax that assigned the exception instance to a variable name by following the Exception type to be caught with a comma.

Here's an example of simple usage:

import sys

try:
    mainstuff()
except (KeyboardInterrupt, EOFError): # the parens are necessary
    sys.exit(0)

I'm specifying only these exceptions to avoid hiding bugs, which if I encounter I expect the full stack trace from.

This is documented here: https://docs.python.org/tutorial/errors.html

You can assign the exception to a variable, (e is common, but you might prefer a more verbose variable if you have long exception handling or your IDE only highlights selections larger than that, as mine does.) The instance has an args attribute. Here is an example:

import sys

try:
    mainstuff()
except (KeyboardInterrupt, EOFError) as err: 
    print(err)
    print(err.args)
    sys.exit(0)

Note that in Python 3, the err object falls out of scope when the except block is concluded.

Deprecated

You may see code that assigns the error with a comma. This usage, the only form available in Python 2.5 and earlier, is deprecated, and if you wish your code to be forward compatible in Python 3, you should update the syntax to use the new form:

import sys

try:
    mainstuff()
except (KeyboardInterrupt, EOFError), err: # don't do this in Python 2.6+
    print err
    print err.args
    sys.exit(0)

If you see the comma name assignment in your codebase, and you're using Python 2.5 or higher, switch to the new way of doing it so your code remains compatible when you upgrade.

The suppress context manager

The accepted answer is really 4 lines of code, minimum:

try:
    do_something()
except (IDontLikeYouException, YouAreBeingMeanException) as e:
    pass

The try, except, pass lines can be handled in a single line with the suppress context manager, available in Python 3.4:

from contextlib import suppress

with suppress(IDontLikeYouException, YouAreBeingMeanException):
     do_something()

So when you want to pass on certain exceptions, use suppress.

Call a stored procedure with another in Oracle

@Michael Lockwood - you don't need to use the keyword "CALL" anywhere. You just need to mention the procedure call directly.

That is

Begin
   proc1(input1, input2);
end;
/

instead of

Begin
   call proc1(input1, input2);
end;
/

"multiple target patterns" Makefile error

Besides having to escape colons as in the original answer, I have found if the indentation is off you could potentially get the same problem. In one makefile, I had to replace spaces with a tab and that allowed me to get past the error.

UICollectionView current visible cell index

For completeness sake, this is the method that ended up working for me. It was a combination of @Anthony & @iAn's methods.

- (void)scrollViewDidEndDecelerating:(UIScrollView *)scrollView {
      CGRect visibleRect = (CGRect){.origin = self.collectionView.contentOffset, .size = self.collectionView.bounds.size};
      CGPoint visiblePoint = CGPointMake(CGRectGetMidX(visibleRect), CGRectGetMidY(visibleRect));
      NSIndexPath *visibleIndexPath = [self.collectionView indexPathForItemAtPoint:visiblePoint];
      NSLog(@"%@",visibleIndexPath);
}

Read file line by line in PowerShell

I was able to read a 4GB log file in about 50 seconds with the following. You may be able to make it faster by loading it as a C# assembly dynamically using PowerShell.

[System.IO.StreamReader]$sr = [System.IO.File]::Open($file, [System.IO.FileMode]::Open)
while (-not $sr.EndOfStream){
    $line = $sr.ReadLine()
}
$sr.Close() 

How to make an executable JAR file?

A jar file is simply a file containing a collection of java files. To make a jar file executable, you need to specify where the main Class is in the jar file. Example code would be as follows.

public class JarExample {

    public static void main(String[] args) {
        javax.swing.SwingUtilities.invokeLater(new Runnable() {
            public void run() {
                // your logic here
            }
        });
    }
}

Compile your classes. To make a jar, you also need to create a Manifest File (MANIFEST.MF). For example,

Manifest-Version: 1.0
Main-Class: JarExample

Place the compiled output class files (JarExample.class,JarExample$1.class) and the manifest file in the same folder. In the command prompt, go to the folder where your files placed, and create the jar using jar command. For example (if you name your manifest file as jexample.mf)

jar cfm jarexample.jar jexample.mf *.class

It will create executable jarexample.jar.

Check an integer value is Null in c#

As stated above, ?? is the null coalescing operator. So the equivalent to

(Age ?? 0) == 0

without using the ?? operator is

(!Age.HasValue) || Age == 0

However, there is no version of .Net that has Nullable< T > but not ??, so your statement,

Now i have to check in a older application where the declaration part is not in ternary.

is doubly invalid.

Going from MM/DD/YYYY to DD-MMM-YYYY in java

Use a SimpleDateFormat to parse the date and then print it out with a SimpleDateFormat withe the desired format.

Here's some code:

    SimpleDateFormat format1 = new SimpleDateFormat("MM/dd/yyyy");
    SimpleDateFormat format2 = new SimpleDateFormat("dd-MMM-yy");
    Date date = format1.parse("05/01/1999");
    System.out.println(format2.format(date));

Output:

01-May-99

How to read multiple Integer values from a single line of input in Java?

Better get the whole line as a string and then use StringTokenizer to get the numbers (using space as delimiter ) and then parse them as integers . This will work for n number of integers in a line .

    Scanner sc = new Scanner(System.in);
    List<Integer> l = new LinkedList<>(); // use linkedlist to save order of insertion
    StringTokenizer st = new StringTokenizer(sc.nextLine(), " "); // whitespace is the delimiter to create tokens
    while(st.hasMoreTokens())  // iterate until no more tokens
    {
        l.add(Integer.parseInt(st.nextToken()));  // parse each token to integer and add to linkedlist

    }

How to write connection string in web.config file and read from it?

Try to use WebConfigurationManager instead of ConfigurationManager

How do you read a file into a list in Python?

hdl = open("C:/name/MyDocuments/numbers", 'r')
milist = hdl.readlines()
hdl.close()

How to force two figures to stay on the same page in LaTeX?

If you want them both on the same page and they'll both take up basically the whole page, then the best idea is to tell LaTeX to put them both on a page of their own!

\begin{figure}[p]

It would probably be against sound typographic principles (e.g., ugly) to have two figures on a page with only a few lines of text above or below them.


By the way, the reason that [!h] works is because it's telling LaTeX to override its usual restrictions on how much space should be devoted to floats on a page with text. As implied above, there's a reason the restrictions are there. Which isn't to say they can be loosened somewhat; see the FAQ on doing that.

How to append strings using sprintf?

Use strcat http://www.cplusplus.com/reference/cstring/strcat/

int main ()
    {
      char str[80];
      strcpy (str,"these ");
      strcat (str,"strings ");
      strcat (str,"are ");
      strcat (str,"concatenated.");
      puts (str);
      return 0;
    }




    Output:


    these strings are concatenated. 

How do I bind a WPF DataGrid to a variable number of columns?

You might be able to do this with AutoGenerateColumns and a DataTemplate. I'm not positive if it would work without a lot of work, you would have to play around with it. Honestly if you have a working solution already I wouldn't make the change just yet unless there's a big reason. The DataGrid control is getting very good but it still needs some work (and I have a lot of learning left to do) to be able to do dynamic tasks like this easily.

Downloading all maven dependencies to a directory NOT in repository?

Please check if you have some config files in ${MAVEN_HOME}/conf directory like settings.xml. Those files overrides settings from .m2 folder and because of that, repository folder from .m2 might not be visible or discarded.

Performance of Arrays vs. Lists

[See also this question]

I've modified Marc's answer to use actual random numbers and actually do the same work in all cases.

Results:

         for      foreach
Array : 1575ms     1575ms (+0%)
List  : 1630ms     2627ms (+61%)
         (+3%)     (+67%)

(Checksum: -1000038876)

Compiled as Release under VS 2008 SP1. Running without debugging on a [email protected], .NET 3.5 SP1.

Code:

class Program
{
    static void Main(string[] args)
    {
        List<int> list = new List<int>(6000000);
        Random rand = new Random(1);
        for (int i = 0; i < 6000000; i++)
        {
            list.Add(rand.Next());
        }
        int[] arr = list.ToArray();

        int chk = 0;
        Stopwatch watch = Stopwatch.StartNew();
        for (int rpt = 0; rpt < 100; rpt++)
        {
            int len = list.Count;
            for (int i = 0; i < len; i++)
            {
                chk += list[i];
            }
        }
        watch.Stop();
        Console.WriteLine("List/for: {0}ms ({1})", watch.ElapsedMilliseconds, chk);

        chk = 0;
        watch = Stopwatch.StartNew();
        for (int rpt = 0; rpt < 100; rpt++)
        {
            int len = arr.Length;
            for (int i = 0; i < len; i++)
            {
                chk += arr[i];
            }
        }
        watch.Stop();
        Console.WriteLine("Array/for: {0}ms ({1})", watch.ElapsedMilliseconds, chk);

        chk = 0;
        watch = Stopwatch.StartNew();
        for (int rpt = 0; rpt < 100; rpt++)
        {
            foreach (int i in list)
            {
                chk += i;
            }
        }
        watch.Stop();
        Console.WriteLine("List/foreach: {0}ms ({1})", watch.ElapsedMilliseconds, chk);

        chk = 0;
        watch = Stopwatch.StartNew();
        for (int rpt = 0; rpt < 100; rpt++)
        {
            foreach (int i in arr)
            {
                chk += i;
            }
        }
        watch.Stop();
        Console.WriteLine("Array/foreach: {0}ms ({1})", watch.ElapsedMilliseconds, chk);
        Console.WriteLine();

        Console.ReadLine();
    }
}

How to show first commit by 'git log'?

You can just reverse your log and just head it for the first result.

git log --pretty=oneline --reverse | head -1

How do I delete an item or object from an array using ng-click?

I disagree that you should be calling a method on your controller. You should be using a service for any actual functionality, and you should be defining directives for any functionality for scalability and modularity, as well as assigning a click event which contains a call to the service which you inject into your directive.

So, for instance, on your HTML...

<a class="btn" ng-remove-birthday="$index">Delete</a>

Then, create a directive...

angular.module('myApp').directive('ngRemoveBirthday', ['myService', function(myService){
    return function(scope, element, attrs){
        angular.element(element.bind('click', function(){
            myService.removeBirthday(scope.$eval(attrs.ngRemoveBirthday), scope);  
        };       
    };
}])

Then in your service...

angular.module('myApp').factory('myService', [function(){
    return {
        removeBirthday: function(birthdayIndex, scope){
            scope.bdays.splice(birthdayIndex);
            scope.$apply();
        }
    };
}]);

When you write your code properly like this, you will make it very easy to write future changes without having to restructure your code. It's organized properly, and you're handling custom click events correctly by binding using custom directives.

For instance, if your client says, "hey, now let's make it call the server and make bread, and then popup a modal." You will be able to easily just go to the service itself without having to add or change any of the HTML, and/or controller method code. If you had just the one line on the controller, you'd eventually need to use a service, for extending the functionality to the heavier lifting the client is asking for.

Also, if you need another 'Delete' button elsewhere, you now have a directive attribute ('ng-remove-birthday') you can easily assign to any element on the page. This now makes it modular and reusable. This will come in handy when dealing with the HEAVY web components paradigm of Angular 2.0. There IS no controller in 2.0. :)

Happy Developing!!!

Increasing the Command Timeout for SQL command

Setting CommandTimeout to 120 is not recommended. Try using pagination as mentioned above. Setting CommandTimeout to 30 is considered as normal. Anything more than that is consider bad approach and that usually concludes something wrong with the Implementation. Now the world is running on MiliSeconds Approach.

Use of PUT vs PATCH methods in REST API real life scenarios

Everyone else has answered the PUT vs PATCH. I was just going to answer what part of the title of the original question asks: "... in REST API real life scenarios". In the real world, this happened to me with internet application that had a RESTful server and a relational database with a Customer table that was "wide" (about 40 columns). I mistakenly used PUT but had assumed it was like a SQL Update command and had not filled out all the columns. Problems: 1) Some columns were optional (so blank was valid answer), 2) many columns rarely changed, 3) some columns the user was not allowed to change such as time stamp of Last Purchase Date, 4) one column was a free-form text "Comments" column that users diligently filled with half-page customer services comments like spouses name to ask about OR usual order, 5) I was working on an internet app at time and there was worry about packet size.

The disadvantage of PUT is that it forces you to send a large packet of info (all columns including the entire Comments column, even though only a few things changed) AND multi-user issue of 2+ users editing the same customer simultaneously (so last one to press Update wins). The disadvantage of PATCH is that you have to keep track on the view/screen side of what changed and have some intelligence to send only the parts that changed. Patch's multi-user issue is limited to editing the same column(s) of same customer.

Java : Sort integer array without using Arrays.sort()

Simple way :

int a[]={6,2,5,1};
            System.out.println(Arrays.toString(a));
             int temp;
             for(int i=0;i<a.length-1;i++){
                 for(int j=0;j<a.length-1;j++){
                     if(a[j] > a[j+1]){   // use < for Descending order
                         temp = a[j+1];
                         a[j+1] = a[j];
                         a[j]=temp;
                     }
                 }
             }
            System.out.println(Arrays.toString(a));

    Output:
    [6, 2, 5, 1]
    [1, 2, 5, 6]

How do you implement a circular buffer in C?

First, the headline. You don't need modulo arithmetic to wrap the buffer if you use bit ints to hold the head & tail "pointers", and size them so they are perfectly in synch. IE: 4096 stuffed into a 12-bit unsigned int is 0 all by itself, unmolested in any way. Eliminating modulo arithmetic, even for powers of 2, doubles the speed - almost exactly.

10 million iterations of filling and draining a 4096 buffer of any type of data elements takes 52 seconds on my 3rd Gen i7 Dell XPS 8500 using Visual Studio 2010's C++ compiler with default inlining, and 1/8192nd of that to service a datum.

I'd RX rewriting the test loops in main() so they no longer control the flow - which is, and should be, controlled by the return values indicating the buffer is full or empty, and the attendant break; statements. IE: the filler and drainer should be able to bang against each other without corruption or instability. At some point I hope to multi-thread this code, whereupon that behavior will be crucial.

The QUEUE_DESC (queue descriptor) and initialization function forces all buffers in this code to be a power of 2. The above scheme will NOT work otherwise. While on the subject, note that QUEUE_DESC is not hard-coded, it uses a manifest constant (#define BITS_ELE_KNT) for its construction. (I'm assuming a power of 2 is sufficient flexibility here)

To make the buffer size run-time selectable, I tried different approaches (not shown here), and settled on using USHRTs for Head, Tail, EleKnt capable of managing a FIFO buffer[USHRT]. To avoid modulo arithmetic I created a mask to && with Head, Tail, but that mask turns out to be (EleKnt -1), so just use that. Using USHRTS instead of bit ints increased performance ~ 15% on a quiet machine. Intel CPU cores have always been faster than their buses, so on a busy, shared machine, packing your data structures gets you loaded and executing ahead of other, competing threads. Trade-offs.

Note the actual storage for the buffer is allocated on the heap with calloc(), and the pointer is at the base of the struct, so the struct and the pointer have EXACTLY the same address. IE; no offset required to be added to the struct address to tie up registers.

In that same vein, all of the variables attendant with servicing the buffer are physically adjacent to the buffer, bound into the same struct, so the compiler can make beautiful assembly language. You'll have to kill the inline optimization to see any assembly, because otherwise it gets crushed into oblivion.

To support the polymorphism of any data type, I've used memcpy() instead of assignments. If you only need the flexibility to support one random variable type per compile, then this code works perfectly.

For polymorphism, you just need to know the type and it's storage requirement. The DATA_DESC array of descriptors provides a way to keep track of each datum that gets put in QUEUE_DESC.pBuffer so it can be retrieved properly. I'd just allocate enough pBuffer memory to hold all of the elements of the largest data type, but keep track of how much of that storage a given datum is actually using in DATA_DESC.dBytes. The alternative is to reinvent a heap manager.

This means QUEUE_DESC's UCHAR *pBuffer would have a parallel companion array to keep track of data type, and size, while a datum's storage location in pBuffer would remain just as it is now. The new member would be something like DATA_DESC *pDataDesc, or, perhaps, DATA_DESC DataDesc[2^BITS_ELE_KNT] if you can find a way to beat your compiler into submission with such a forward reference. Calloc() is always more flexible in these situations.

You'd still memcpy() in Q_Put(),Q_Get, but the number of bytes actually copied would be determined by DATA_DESC.dBytes, not QUEUE_DESC.EleBytes. The elements are potentially all of different types/sizes for any given put or get.

I believe this code satisfies the speed and buffer size requirements, and can be made to satisfy the requirement for 6 different data types. I've left the many test fixtures in, in the form of printf() statements, so you can satisfy yourself (or not) that the code works properly. The random number generator demonstrates that the code works for any random head/tail combo.

enter code here
// Queue_Small.cpp : Defines the entry point for the console application.
//
#include "stdafx.h"
#include <stdio.h>
#include <time.h>
#include <limits.h>
#include <stdlib.h>
#include <malloc.h>
#include <memory.h>
#include <math.h>

#define UCHAR unsigned char
#define ULONG unsigned long
#define USHRT unsigned short
#define dbl   double
/* Queue structure */
#define QUEUE_FULL_FLAG 1
#define QUEUE_EMPTY_FLAG -1
#define QUEUE_OK 0
//  
#define BITS_ELE_KNT    12  //12 bits will create 4.096 elements numbered 0-4095
//
//typedef struct    {
//  USHRT dBytes:8;     //amount of QUEUE_DESC.EleBytes storage used by datatype
//  USHRT dType :3; //supports 8 possible data types (0-7)
//  USHRT dFoo  :5; //unused bits of the unsigned short host's storage
// }    DATA_DESC;
//  This descriptor gives a home to all the housekeeping variables
typedef struct  {
    UCHAR   *pBuffer;   //  pointer to storage, 16 to 4096 elements
    ULONG Tail  :BITS_ELE_KNT;  //  # elements, with range of 0-4095
    ULONG Head  :BITS_ELE_KNT;  //  # elements, with range of 0-4095
    ULONG EleBytes  :8;     //  sizeof(elements) with range of 0-256 bytes
    // some unused bits will be left over if BITS_ELE_KNT < 12
    USHRT EleKnt    :BITS_ELE_KNT +1;// 1 extra bit for # elements (1-4096)
    //USHRT Flags   :(8*sizeof(USHRT) - BITS_ELE_KNT +1);   //  flags you can use
    USHRT   IsFull  :1;     // queue is full
    USHRT   IsEmpty :1;     // queue is empty
    USHRT   Unused  :1;     // 16th bit of USHRT
}   QUEUE_DESC;

//  ---------------------------------------------------------------------------
//  Function prototypes
QUEUE_DESC *Q_Init(QUEUE_DESC *Q, int BitsForEleKnt, int DataTypeSz);
int Q_Put(QUEUE_DESC *Q, UCHAR *pNew);
int Q_Get(UCHAR *pOld, QUEUE_DESC *Q);
//  ---------------------------------------------------------------------------
QUEUE_DESC *Q_Init(QUEUE_DESC *Q, int BitsForEleKnt, int DataTypeSz)    {
    memset((void *)Q, 0, sizeof(QUEUE_DESC));//init flags and bit integers to zero
    //select buffer size from powers of 2 to receive modulo 
    //                arithmetic benefit of bit uints overflowing
    Q->EleKnt   =   (USHRT)pow(2.0, BitsForEleKnt);
    Q->EleBytes =   DataTypeSz; // how much storage for each element?
    //  Randomly generated head, tail a test fixture only. 
    //      Demonstrates that the queue can be entered at a random point 
    //      and still perform properly. Normally zero
    srand(unsigned(time(NULL)));    // seed random number generator with current time
    Q->Head = Q->Tail = rand(); // supposed to be set to zero here, or by memset
    Q->Head = Q->Tail = 0;
    //  allocate queue's storage
    if(NULL == (Q->pBuffer = (UCHAR *)calloc(Q->EleKnt, Q->EleBytes)))  {
        return NULL;
    }   else    {
        return Q;
    }
}
//  ---------------------------------------------------------------------------
int Q_Put(QUEUE_DESC *Q, UCHAR *pNew)   
{
    memcpy(Q->pBuffer + (Q->Tail * Q->EleBytes), pNew, Q->EleBytes);
    if(Q->Tail == (Q->Head + Q->EleKnt)) {
        //  Q->IsFull = 1;
        Q->Tail += 1;   
        return QUEUE_FULL_FLAG; //  queue is full
    }
    Q->Tail += 1;   //  the unsigned bit int MUST wrap around, just like modulo
    return QUEUE_OK; // No errors
}
//  ---------------------------------------------------------------------------
int Q_Get(UCHAR *pOld, QUEUE_DESC *Q)   
{
    memcpy(pOld, Q->pBuffer + (Q->Head * Q->EleBytes), Q->EleBytes);
    Q->Head += 1;   //  the bit int MUST wrap around, just like modulo

    if(Q->Head == Q->Tail)      {
        //  Q->IsEmpty = 1;
        return QUEUE_EMPTY_FLAG; // queue Empty - nothing to get
    }
    return QUEUE_OK; // No errors
}
//
//  ---------------------------------------------------------------------------
int _tmain(int argc, _TCHAR* argv[])    {
//  constrain buffer size to some power of 2 to force faux modulo arithmetic
    int LoopKnt = 1000000;  //  for benchmarking purposes only
    int k, i=0, Qview=0;
    time_t start;
    QUEUE_DESC Queue, *Q;
    if(NULL == (Q = Q_Init(&Queue, BITS_ELE_KNT, sizeof(int)))) {
        printf("\nProgram failed to initialize. Aborting.\n\n");
        return 0;
    }

    start = clock();
    for(k=0; k<LoopKnt; k++)    {
        //printf("\n\n Fill'er up please...\n");
        //Q->Head = Q->Tail = rand();
        for(i=1; i<= Q->EleKnt; i++)    {
            Qview = i*i;
            if(QUEUE_FULL_FLAG == Q_Put(Q, (UCHAR *)&Qview))    {
                //printf("\nQueue is full at %i \n", i);
                //printf("\nQueue value of %i should be %i squared", Qview, i);
                break;
            }
            //printf("\nQueue value of %i should be %i squared", Qview, i);
        }
        //  Get data from queue until completely drained (empty)
        //
        //printf("\n\n Step into the lab, and see what's on the slab... \n");
        Qview = 0;
        for(i=1; i; i++)    {
            if(QUEUE_EMPTY_FLAG == Q_Get((UCHAR *)&Qview, Q))   {
                //printf("\nQueue value of %i should be %i squared", Qview, i);
                //printf("\nQueue is empty at %i", i);
                break;
            }
            //printf("\nQueue value of %i should be %i squared", Qview, i);
        }
        //printf("\nQueue head value is %i, tail is %i\n", Q->Head, Q->Tail);
    }
    printf("\nQueue time was %5.3f to fill & drain %i element queue  %i times \n", 
                     (dbl)(clock()-start)/(dbl)CLOCKS_PER_SEC,Q->EleKnt, LoopKnt);
    printf("\nQueue head value is %i, tail is %i\n", Q->Head, Q->Tail);
    getchar();
    return 0;
}

A server is already running. Check …/tmp/pids/server.pid. Exiting - rails

the gui way for Windows user

open the ResourceMonitor (taskmanager ->Performance -> ResourceMonitor) and kill the ruby.exe process

enter image description here

What is the difference between 'SAME' and 'VALID' padding in tf.nn.max_pool of tensorflow?

To sum up, 'valid' padding means no padding. The output size of the convolutional layer shrinks depending on the input size & kernel size.

On the contrary, 'same' padding means using padding. When the stride is set as 1, the output size of the convolutional layer maintains as the input size by appending a certain number of '0-border' around the input data when calculating convolution.

Hope this intuitive description helps.

How can I jump to class/method definition in Atom text editor?

The functionality is already present in atom via the Symbols View package you don't need to install anything.

The command you are searching for is symbols-view:go-to-declaration (Jump to the symbol under the cursor) which is bound by default to cmd-alt-down on macOS and ctrl-alt-down on Linux.

just note that it will work only if you will have generated tags for your project, either via this package or via ctags (exuberant or not)

Java - Using Accessor and Mutator methods

You need to remove the static from your accessor methods - these methods need to be instance methods and access the instance variables

public class IDCard {
    public String name, fileName;
    public int id;

    public IDCard(final String name, final String fileName, final int id) {
        this.name = name;
        this.fileName = fileName
        this.id = id;
    }

    public String getName() {
        return name;
    }
}

You can the create an IDCard and use the accessor like this:

final IDCard card = new IDCard();
card.getName();

Each time you call new a new instance of the IDCard will be created and it will have it's own copies of the 3 variables.

If you use the static keyword then those variables are common across every instance of IDCard.

A couple of things to bear in mind:

  1. don't add useless comments - they add code clutter and nothing else.
  2. conform to naming conventions, use lower case of variable names - name not Name.

How to get the first word of a sentence in PHP?

All of the answers here are using an approach that the processor needs to search all of the string even if the first word is found! For big strings, this is not recommended. This approach is optimal:

function getFirstWord($string) {
    $result = "";
    foreach($string as $char) {
        if($char == " " && strlen($result)) {
            break;
        }
        $result .= $char;
    }
    return $result;
}

Get the element with the highest occurrence in an array

I came up with a shorter solution, but it's using lodash. Works with any data, not just strings. For objects can be used:

const mostFrequent = _.maxBy(Object.values(_.groupBy(inputArr, el => el.someUniqueProp)), arr => arr.length)[0];

This is for strings:

const mostFrequent = _.maxBy(Object.values(_.groupBy(inputArr, el => el)), arr => arr.length)[0];

Just grouping data under a certain criteria, then finding the largest group.

Using FileUtils in eclipse

For selenium automation users

  1. Download Library file from http://www.java2s.com/Code/Jar/o/Downloadorgapachecommonsiojar.htm
  2. Extract
  3. Right click on the proj name from the explorer >> Build path >>Config Build Path

Accessing dict keys like an attribute?

You can have all legal string characters as part of the key if you use array notation. For example, obj['!#$%^&*()_']

"dd/mm/yyyy" date format in excel through vba

I got it

Cells(1, 1).Value = StartDate
Cells(1, 1).NumberFormat = "dd/mm/yyyy"

Basically, I need to set the cell format, instead of setting the date.

Spring MVC: How to return image in @ResponseBody?

In addition to registering a ByteArrayHttpMessageConverter, you may want to use a ResponseEntity instead of @ResponseBody. The following code works for me :

@RequestMapping("/photo2")
public ResponseEntity<byte[]> testphoto() throws IOException {
    InputStream in = servletContext.getResourceAsStream("/images/no_image.jpg");

    final HttpHeaders headers = new HttpHeaders();
    headers.setContentType(MediaType.IMAGE_PNG);

    return new ResponseEntity<byte[]>(IOUtils.toByteArray(in), headers, HttpStatus.CREATED);
}

Opening A Specific File With A Batch File?

@echo off
cd "folder directory to your file"
start filename.ext

For example:

cd "C:\Program Files (x86)\Winamp" 
Start winamp.exe

Set a button background image iPhone programmatically

This will work

UIImage *buttonImage = [UIImage imageNamed:@"imageName.png"];
[btn setImage:buttonImage forState:UIControlStateNormal];
[self.view addSubview:btn];

How can I get current date in Android?

This is nothing to do with android as it is java based so you could use

private String getDateTime() { 
   DateFormat dateFormat = new SimpleDateFormat("yyyy/MM/dd HH:mm:ss");
   Date date = new Date(); 
   return dateFormat.format(date); 
}

SQLite - UPSERT *not* INSERT or REPLACE

The best approach I know is to do an update, followed by an insert. The "overhead of a select" is necessary, but it is not a terrible burden since you are searching on the primary key, which is fast.

You should be able to modify the below statements with your table & field names to do what you want.

--first, update any matches
UPDATE DESTINATION_TABLE DT
SET
  MY_FIELD1 = (
              SELECT MY_FIELD1
              FROM SOURCE_TABLE ST
              WHERE ST.PRIMARY_KEY = DT.PRIMARY_KEY
              )
 ,MY_FIELD2 = (
              SELECT MY_FIELD2
              FROM SOURCE_TABLE ST
              WHERE ST.PRIMARY_KEY = DT.PRIMARY_KEY
              )
WHERE EXISTS(
            SELECT ST2.PRIMARY_KEY
            FROM
              SOURCE_TABLE ST2
             ,DESTINATION_TABLE DT2
            WHERE ST2.PRIMARY_KEY = DT2.PRIMARY_KEY
            );

--second, insert any non-matches
INSERT INTO DESTINATION_TABLE(
  MY_FIELD1
 ,MY_FIELD2
)
SELECT
  ST.MY_FIELD1
 ,NULL AS MY_FIELD2  --insert NULL into this field
FROM
  SOURCE_TABLE ST
WHERE NOT EXISTS(
                SELECT DT2.PRIMARY_KEY
                FROM DESTINATION_TABLE DT2
                WHERE DT2.PRIMARY_KEY = ST.PRIMARY_KEY
                );

How do I find the absolute position of an element using jQuery?

.offset() will return the offset position of an element as a simple object, eg:

var position = $(element).offset(); // position = { left: 42, top: 567 }

You can use this return value to position other elements at the same spot:

$(anotherElement).css(position)

How to insert element into arrays at specific position?

array_slice() can be used to extract parts of the array, and the union array operator (+) can recombine the parts.

$res = array_slice($array, 0, 3, true) +
    array("my_key" => "my_value") +
    array_slice($array, 3, count($array)-3, true);

This example:

$array = array(
  'zero'  => '0',
  'one'   => '1',
  'two'   => '2',
  'three' => '3',
);
$res = array_slice($array, 0, 3, true) +
    array("my_key" => "my_value") +
    array_slice($array, 3, count($array) - 1, true) ;
print_r($res);

gives:

Array
(
    [zero] => 0
    [one] => 1
    [two] => 2
    [my_key] => my_value
    [three] => 3
)

Android Crop Center of Bitmap

public Bitmap getResizedBitmap(Bitmap bm) {
    int width = bm.getWidth();
    int height = bm.getHeight();

    int narrowSize = Math.min(width, height);
    int differ = (int)Math.abs((bm.getHeight() - bm.getWidth())/2.0f);
    width  = (width  == narrowSize) ? 0 : differ;
    height = (width == 0) ? differ : 0;

    Bitmap resizedBitmap = Bitmap.createBitmap(bm, width, height, narrowSize, narrowSize);
    bm.recycle();
    return resizedBitmap;
}

echo that outputs to stderr

My suggestion:

echo "my errz" >> /proc/self/fd/2

or

echo "my errz" >> /dev/stderr

echo "my errz" > /proc/self/fd/2 will effectively output to stderr because /proc/self is a link to the current process, and /proc/self/fd holds the process opened file descriptors, and then, 0, 1, and 2 stand for stdin, stdout and stderr respectively.

The /proc/self link doesn't work on MacOS, however, /proc/self/fd/* is available on Termux on Android, but not /dev/stderr. How to detect the OS from a Bash script? can help if you need to make your script more portable by determining which variant to use.

Why is this jQuery click function not working?

I found the best solution for this problem by using ON with $(document).

 $(document).on('click', '#yourid', function() { alert("hello"); });

for id start with see below:

$(document).on('click', 'div[id^="start"]', function() {
alert ('hello'); });

finally after 1 week I not need to add onclick triger. I hope this will help many people

the best way to make codeigniter website multi-language. calling from lang arrays depends on lang session?

In the controller add following lines when you make the cunstructor

i.e, after

parent::Controller();

add below lines

    $this->load->helper('lang_translate');
    $this->lang->load('nl_site', 'nl'); // ('filename', 'directory')

create helper file lang_translate_helper.php with following function and put it in directory system\application\helpers

function label($label, $obj)
{
    $return = $obj->lang->line($label);
    if($return)
        echo $return;
    else
        echo $label;
}

for each of the language, create a directory with language abbrevation like en, nl, fr, etc., under system\application\languages

create language file in above (respective) directory which will contain $lang array holding pairs label=>language_value as given below

nl_site_lang.php

$lang['welcome'] = 'Welkom';
$lang['hello word'] = 'worde Witaj';

en_site_lang.php

$lang['welcome'] = 'Welcome';
$lang['hello word'] = 'Hello Word';

you can store multiple files for same language with differently as per the requirement e.g, if you want separate language file for managing backend (administrator section) you can use it in controller as $this->lang->load('nl_admin', 'nl');

nl_admin_lang.php

$lang['welcome'] = 'Welkom';
$lang['hello word'] = 'worde Witaj';

and finally to print the label in desired language, access labels as below in view

label('welcome', $this);

OR

label('hello word', $this);

note the space in hello & word you can use it like this way as well :)

whene there is no lable defined in the language file, it will simply print it what you passed to the function label.

Javascript: Call a function after specific time period

sounds like you're looking for setInterval. It's as easy as this:

function FetchData() {
  // do something
}
setInterval(FetchData, 60000);

if you only want to call something once, theres setTimeout.

Importing json file in TypeScript

With TypeScript 2.9.+ you can simply import JSON files with typesafety and intellisense like this:

import colorsJson from '../colors.json'; // This import style requires "esModuleInterop", see "side notes"
console.log(colorsJson.primaryBright);

Make sure to add these settings in the compilerOptions section of your tsconfig.json (documentation):

"resolveJsonModule": true,
"esModuleInterop": true,

Side notes:

  • Typescript 2.9.0 has a bug with this JSON feature, it was fixed with 2.9.2
  • The esModuleInterop is only necessary for the default import of the colorsJson. If you leave it set to false then you have to import it with import * as colorsJson from '../colors.json'

Pylint, PyChecker or PyFlakes?

Well, I am a bit curious, so I just tested the three myself right after asking the question ;-)

Ok, this is not a very serious review, but here is what I can say:

I tried the tools with the default settings (it's important because you can pretty much choose your check rules) on the following script:

#!/usr/local/bin/python
# by Daniel Rosengren modified by e-satis

import sys, time
stdout = sys.stdout

BAILOUT = 16
MAX_ITERATIONS = 1000

class Iterator(object) :

    def __init__(self):

        print 'Rendering...'
        for y in xrange(-39, 39):
            stdout.write('\n')
            for x in xrange(-39, 39):
                if self.mandelbrot(x/40.0, y/40.0) :
                    stdout.write(' ')
                else:
                    stdout.write('*')


    def mandelbrot(self, x, y):
        cr = y - 0.5
        ci = x
        zi = 0.0
        zr = 0.0

        for i in xrange(MAX_ITERATIONS) :
            temp = zr * zi
            zr2 = zr * zr
            zi2 = zi * zi
            zr = zr2 - zi2 + cr
            zi = temp + temp + ci

            if zi2 + zr2 > BAILOUT:
                return i

        return 0

t = time.time()
Iterator()
print '\nPython Elapsed %.02f' % (time.time() - t)

As a result:

  • PyChecker is troublesome because it compiles the module to analyze it. If you don't want your code to run (e.g, it performs a SQL query), that's bad.
  • PyFlakes is supposed to be light. Indeed, it decided that the code was perfect. I am looking for something quite severe so I don't think I'll go for it.
  • PyLint has been very talkative and rated the code 3/10 (OMG, I'm a dirty coder !).

Strong points of PyLint:

  • Very descriptive and accurate report.
  • Detect some code smells. Here it told me to drop my class to write something with functions because the OO approach was useless in this specific case. Something I knew, but never expected a computer to tell me :-p
  • The fully corrected code run faster (no class, no reference binding...).
  • Made by a French team. OK, it's not a plus for everybody, but I like it ;-)

Cons of Pylint:

  • Some rules are really strict. I know that you can change it and that the default is to match PEP8, but is it such a crime to write 'for x in seq'? Apparently yes because you can't write a variable name with less than 3 letters. I will change that.
  • Very very talkative. Be ready to use your eyes.

Corrected script (with lazy doc strings and variable names):

#!/usr/local/bin/python
# by Daniel Rosengren, modified by e-satis
"""
Module doctring
"""


import time
from sys import stdout

BAILOUT = 16
MAX_ITERATIONS = 1000

def mandelbrot(dim_1, dim_2):
    """
    function doc string
    """
    cr1 = dim_1 - 0.5
    ci1 = dim_2
    zi1 = 0.0
    zr1 = 0.0

    for i in xrange(MAX_ITERATIONS) :
        temp = zr1 * zi1
        zr2 = zr1 * zr1
        zi2 = zi1 * zi1
        zr1 = zr2 - zi2 + cr1
        zi1 = temp + temp + ci1

        if zi2 + zr2 > BAILOUT:
            return i

    return 0

def execute() :
    """
    func doc string
    """
    print 'Rendering...'
    for dim_1 in xrange(-39, 39):
        stdout.write('\n')
        for dim_2 in xrange(-39, 39):
            if mandelbrot(dim_1/40.0, dim_2/40.0) :
                stdout.write(' ')
            else:
                stdout.write('*')


START_TIME = time.time()
execute()
print '\nPython Elapsed %.02f' % (time.time() - START_TIME)

Thanks to Rudiger Wolf, I discovered pep8 that does exactly what its name suggests: matching PEP8. It has found several syntax no-nos that Pylint did not. But Pylint found stuff that was not specifically linked to PEP8 but interesting. Both tools are interesting and complementary.

Eventually I will use both since there are really easy to install (via packages or setuptools) and the output text is so easy to chain.

To give you a little idea of their output:

pep8:

./python_mandelbrot.py:4:11: E401 multiple imports on one line
./python_mandelbrot.py:10:1: E302 expected 2 blank lines, found 1
./python_mandelbrot.py:10:23: E203 whitespace before ':'
./python_mandelbrot.py:15:80: E501 line too long (108 characters)
./python_mandelbrot.py:23:1: W291 trailing whitespace
./python_mandelbrot.py:41:5: E301 expected 1 blank line, found 3

Pylint:

************* Module python_mandelbrot
C: 15: Line too long (108/80)
C: 61: Line too long (85/80)
C:  1: Missing docstring
C:  5: Invalid name "stdout" (should match (([A-Z_][A-Z0-9_]*)|(__.*__))$)
C: 10:Iterator: Missing docstring
C: 15:Iterator.__init__: Invalid name "y" (should match [a-z_][a-z0-9_]{2,30}$)
C: 17:Iterator.__init__: Invalid name "x" (should match [a-z_][a-z0-9_]{2,30}$)

[...] and a very long report with useful stats like :

Duplication
-----------

+-------------------------+------+---------+-----------+
|                         |now   |previous |difference |
+=========================+======+=========+===========+
|nb duplicated lines      |0     |0        |=          |
+-------------------------+------+---------+-----------+
|percent duplicated lines |0.000 |0.000    |=          |
+-------------------------+------+---------+-----------+

Where to find the win32api module for Python?

There is a a new option as well: get it via pip! There is a package pypiwin32 with wheels available, so you can just install with: pip install pypiwin32!

Edit: Per comment from @movermeyer, the main project now publishes wheels at pywin32, and so can be installed with pip install pywin32

Difference between java.lang.RuntimeException and java.lang.Exception

The runtime exception classes (RuntimeException and its subclasses) are exempted from compile-time checking, since the compiler cannot establish that run-time exceptions cannot occur. (from JLS).

In the classes that you design you should subclass Exception and throw instances of it to signal any exceptional scenarios. Doing so you will be explicitly signaling the clients of your class that usage of your class might throw exception and they have to take steps to handle those exceptional scenarios.

Below code snippets explain this point:

//Create your own exception class subclassing from Exception
class MyException extends Exception {
    public MyException(final String message) {
        super(message);
    }
}

public class Process {
    public void execute() {
        throw new RuntimeException("Runtime");
    }  
    public void process() throws MyException {
        throw new MyException("Checked");
    }
}

In the above class definition of class Process, the method execute can throw a RuntimeException but the method declaration need not specify that it throws RuntimeException.

The method process throws a checked exception and it should declare that it will throw a checked exception of kind MyException and not doing so will be a compile error.

The above class definition will affect the code that uses Process class as well.

The call new Process().execute() is a valid invocation where as the call of form new Process().process() gives a compile error. This is because the client code should take steps to handle MyException (say call to process() can be enclosed in a try/catch block).

pip3: command not found

After yum install python3-pip, check the name of the installed binary. e.g.

ll /usr/bin/pip*

On my CentOS 7, it is named as pip-3 instead of pip3.

iOS Safari – How to disable overscroll but allow scrollable divs to scroll normally?

I was looking for a way to prevent all body scrolling when there's a popup with a scrollable area (a "shopping cart" popdown that has a scrollable view of your cart).

I wrote a far more elegant solution using minimal javascript to just toggle the class "noscroll" on your body when you have a popup or div that you'd like to scroll (and not "overscroll" the whole page body).

while desktop browsers observe overflow:hidden -- iOS seems to ignore that unless you set the position to fixed... which causes the whole page to be a strange width, so you have to set the position and width manually as well. use this css:

.noscroll {
    overflow: hidden;
    position: fixed;
    top: 0;
    left: 0;
    width: 100%;
}

and this jquery:

/* fade in/out cart popup, add/remove .noscroll from body */
$('a.cart').click(function() {
    $('nav > ul.cart').fadeToggle(100, 'linear');
    if ($('nav > ul.cart').is(":visible")) {
        $('body').toggleClass('noscroll');
    } else {
        $('body').removeClass('noscroll');
    }
});

/* close all popup menus when you click the page... */
$('body').click(function () {
    $('nav > ul').fadeOut(100, 'linear');
    $('body').removeClass('noscroll');
});

/* ... but prevent clicks in the popup from closing the popup */
$('nav > ul').click(function(event){
    event.stopPropagation();
});

Update multiple rows using select statement

You can use alias to improve the query:

UPDATE t1
   SET t1.Value = t2.Value
  FROM table1 AS t1
         INNER JOIN 
       table2 AS t2
         ON t1.ID = t2.ID

How to create war files

Use ant build code I use this for my project SMS

<property name="WEB-INF" value="${basedir}/WebRoot/WEB-INF" />
<property name="OUT" value="${basedir}/out" />
<property name="WAR_FILE_NAME" value="mywebapplication.war" />
<property name="TEMP" value="${basedir}/temp" />

<target name="help">
    <echo>
        --------------------------------------------------
        compile - Compile
        archive - Generate WAR file
        --------------------------------------------------
    </echo>
</target>

<target name="init">
    <delete dir="${WEB-INF}/classes" />
    <mkdir dir="${WEB-INF}/classes" />
</target>

<target name="compile" depends="init">
    <javac srcdir="${basedir}/src" 
                destdir="${WEB-INF}/classes" 
                classpathref="libs">
    </javac>
</target>

<target name="archive" depends="compile">
    <delete dir="${OUT}" />
    <mkdir dir="${OUT}" />
    <delete dir="${TEMP}" />
    <mkdir dir="${TEMP}" />
    <copy todir="${TEMP}" >
        <fileset dir="${basedir}/WebRoot">
        </fileset>
    </copy>
    <move file="${TEMP}/log4j.properties" 
                    todir="${TEMP}/WEB-INF/classes" />
    <war destfile="${OUT}/${WAR_FILE_NAME}" 
                    basedir="${TEMP}" 
                    compress="true" 
                    webxml="${TEMP}/WEB-INF/web.xml" />
    <delete dir="${TEMP}" />
</target>

<path id="libs">
    <fileset includes="*.jar" dir="${WEB-INF}/lib" />
</path>

MYSQL query between two timestamps

Try:

SELECT * 
FROM eventList 
WHERE  `date` BETWEEN FROM_UNIXTIME(1364256001) AND FROM_UNIXTIME(1364342399)

Or

SELECT * 
FROM eventList WHERE  `date` 
BETWEEN '2013-03-26 00:00:01' AND '2013-03-26 23:59:59'

Calculate distance between 2 GPS coordinates

I guess you want it along the curvature of the earth. Your two points and the center of the earth are on a plane. The center of the earth is the center of a circle on that plane and the two points are (roughly) on the perimeter of that circle. From that you can calculate the distance by finding out what the angle from one point to the other is.

If the points are not the same heights, or if you need to take into account that the earth is not a perfect sphere it gets a little more difficult.

How to convert a data frame column to numeric type?

Though others have covered the topic pretty well, I'd like to add this additional quick thought/hint. You could use regexp to check in advance whether characters potentially consist only of numerics.

for(i in seq_along(names(df)){
     potential_numcol[i] <- all(!grepl("[a-zA-Z]",d[,i]))
}
# and now just convert only the numeric ones
d <- sapply(d[,potential_numcol],as.numeric)

For more sophisticated regular expressions and a neat why to learn/experience their power see this really nice website: http://regexr.com/

Using multiprocessing.Process with a maximum number of simultaneous processes

more generally, this could also look like this:

import multiprocessing
def chunks(l, n):
    for i in range(0, len(l), n):
        yield l[i:i + n]

numberOfThreads = 4


if __name__ == '__main__':
    jobs = []
    for i, param in enumerate(params):
        p = multiprocessing.Process(target=f, args=(i,param))
        jobs.append(p)
    for i in chunks(jobs,numberOfThreads):
        for j in i:
            j.start()
        for j in i:
            j.join()

Of course, that way is quite cruel (since it waits for every process in a junk until it continues with the next chunk). Still it works well for approx equal run times of the function calls.

Gmail Error :The SMTP server requires a secure connection or the client was not authenticated. The server response was: 5.5.1 Authentication Required

When you try to send mail from code and you find the error "The SMTP server requires a secure connection or the client was not authenticated. The server response was: 5.5.1 Authentication Required", than the error might occur due to following cases.

case 1: when the password is wrong

case 2: when you try to login from some App

case 3: when you try to login from the domain other than your time zone/domain/computer (This is the case in most of scenarios when sending mail from code)

There is a solution for each

solution for case 1: Enter the correct password.

solution 1 for case 2: go to security settings at the followig link https://www.google.com/settings/security/lesssecureapps and enable less secure apps . So that you will be able to login from all apps.

solution 2 for case 2:(see https://stackoverflow.com/a/9572958/52277) enable two-factor authentication (aka two-step verification) , and then generate an application-specific password. Use that newly generated password to authenticate via SMTP.

solution 1 for case 3: (This might be helpful) you need to review the activity. but reviewing the activity will not be helpful due to latest security standards the link will not be useful. So try the below case.

solution 2 for case 3: If you have hosted your code somewhere on production server and if you have access to the production server, than take remote desktop connection to the production server and try to login once from the browser of the production server. This will add excpetioon for login to google and you will be allowed to login from code.

But what if you don't have access to the production server. try the solution 3

solution 3 for case 3: You have to enable login from other timezone / ip for your google account.

to do this follow the link https://g.co/allowaccess and allow access by clicking the continue button.

And that's it. Here you go. Now you will be able to login from any of the computer and by any means of app to your google account.

Reading Datetime value From Excel sheet

Another option: when cell type is unknown at compile time and cell is formatted as Date Range.Value returns a desired DateTime object.


public static DateTime? GetAsDateTimeOrDefault(Range cell)
{
    object cellValue = cell.Value;
    if (cellValue is DateTime result)
    {
        return result;
    }
    return null;
}

How to position a DIV in a specific coordinates?

You don't have to use Javascript to do this. Using plain-old css:

div.blah {
  position:absolute;
  top: 0; /*[wherever you want it]*/
  left:0; /*[wherever you want it]*/
}

If you feel you must use javascript, or are trying to do this dynamically Using JQuery, this affects all divs of class "blah":

var blahclass =  $('.blah'); 
blahclass.css('position', 'absolute');
blahclass.css('top', 0); //or wherever you want it
blahclass.css('left', 0); //or wherever you want it

Alternatively, if you must use regular old-javascript you can grab by id

var domElement = document.getElementById('myElement');// don't go to to DOM every time you need it. Instead store in a variable and manipulate.
domElement.style.position = "absolute";
domElement.style.top = 0; //or whatever 
domElement.style.left = 0; // or whatever

How to retrieve element value of XML using Java?

If your XML is a String, Then you can do the following:

String xml = ""; //Populated XML String....

DocumentBuilderFactory factory = DocumentBuilderFactory.newInstance();
DocumentBuilder builder = factory.newDocumentBuilder();
Document document = builder.parse(new InputSource(new StringReader(xml)));
Element rootElement = document.getDocumentElement();

If your XML is in a file, then Document document will be instantiated like this:

Document document = builder.parse(new File("file.xml"));

The document.getDocumentElement() returns you the node that is the document element of the document (in your case <config>).

Once you have a rootElement, you can access the element's attribute (by calling rootElement.getAttribute() method), etc. For more methods on java's org.w3c.dom.Element

More info on java DocumentBuilder & DocumentBuilderFactory. Bear in mind, the example provided creates a XML DOM tree so if you have a huge XML data, the tree can be huge.


Update Here's an example to get "value" of element <requestqueue>

protected String getString(String tagName, Element element) {
        NodeList list = element.getElementsByTagName(tagName);
        if (list != null && list.getLength() > 0) {
            NodeList subList = list.item(0).getChildNodes();

            if (subList != null && subList.getLength() > 0) {
                return subList.item(0).getNodeValue();
            }
        }

        return null;
    }

You can effectively call it as,

String requestQueueName = getString("requestqueue", element);

Bootstrap combining rows (rowspan)

Note: This was for Bootstrap 2 (relevant when the question was asked).

You can accomplish this by using row-fluid to make a fluid (percentage) based row inside an existing block.

<div class="row">
   <div class="span5">span5</div>
   <div class="span3">span3</div>
   <div class="span2">
      <div class="row-fluid">
         <div class="span12">span2</div>
         <div class="span12">span2</div>
      </div>
   </div>
   <div class="span2">span2</div>
</div>
<div class="row">
   <div class="span6">
      <div class="row-fluid">
         <div class="span12">span6</div>
         <div class="span12">span6</div>
      </div>
   </div>
   <div class="span6">span6</div>
</div>

Here's a JSFiddle example.

I did notice that there was an odd left margin that appears (or does not appear) for the spans inside of the row-fluid after the first one. This can be fixed with a small CSS tweak (it's the same CSS that is applied to the first child, expanded to those past the first child):

.row-fluid [class*="span"] {
    margin-left: 0;
}

Possible to view PHP code of a website?

A bug or security vulnerability in the server (either Apache or the PHP engine), or your own PHP code, might allow an attacker to obtain access to your code.

For instance if you have a PHP script to allow people to download files, and an attacker can trick this script into download some of your PHP files, then your code can be leaked.

Since it's impossible to eliminate all bugs from the software you're using, if someone really wants to steal your code, and they have enough resources, there's a reasonable chance they'll be able to.

However, as long as you keep your server up-to-date, someone with casual interest is not able to see the PHP source unless there are some obvious security vulnerabilities in your code.

Read the Security section of the PHP manual as a starting point to keeping your code safe.

XSD - how to allow elements in any order any number of times?

In the schema you have in your question, child1 or child2 can appear in any order, any number of times. So this sounds like what you are looking for.

Edit: if you wanted only one of them to appear an unlimited number of times, the unbounded would have to go on the elements instead:

Edit: Fixed type in XML.

Edit: Capitalised O in maxOccurs

<xs:element name="foo">
   <xs:complexType>
     <xs:choice maxOccurs="unbounded">
       <xs:element name="child1" type="xs:int" maxOccurs="unbounded"/>
       <xs:element name="child2" type="xs:string" maxOccurs="unbounded"/>
     </xs:choice>
   </xs:complexType>
</xs:element>

Hide Twitter Bootstrap nav collapse on click

_x000D_
_x000D_
$('.nav a').click(function () {_x000D_
    $('.navbar-collapse').collapse('hide');_x000D_
});
_x000D_
_x000D_
_x000D_

What is perm space?

It stands for permanent generation:

The permanent generation is special because it holds meta-data describing user classes (classes that are not part of the Java language). Examples of such meta-data are objects describing classes and methods and they are stored in the Permanent Generation. Applications with large code-base can quickly fill up this segment of the heap which will cause java.lang.OutOfMemoryError: PermGen no matter how high your -Xmx and how much memory you have on the machine.

How to get the first day of the current week and month?

public static void main(String[] args) {
    System.out.println(getMonthlyEpochList(1498867199L,12,"Monthly"));

}

public static Map<String,String> getMonthlyEpochList(Long currentEpoch, int noOfTerms, String timeMode) {
    Map<String,String> map = new LinkedHashMap<String,String>();
    int month = 0;
    while(noOfTerms != 0) {
        Calendar calendar = Calendar.getInstance();         
        calendar.add(Calendar.MONTH, month);
        calendar.set(Calendar.DATE, calendar.getActualMinimum(Calendar.DAY_OF_MONTH));
        Date monthFirstDay = calendar.getTime();
        calendar.set(Calendar.DATE, calendar.getActualMaximum(Calendar.DAY_OF_MONTH));
        Date monthLastDay = calendar.getTime();
        map.put(getMMYY(monthFirstDay.getTime()), monthFirstDay + ":" +monthLastDay);
        month--;
        noOfTerms--;
    }
    return map;     
}

Excel 2010 VBA Referencing Specific Cells in other worksheets

I am going to give you a simplistic answer that hopefully will help you with VBA in general. The easiest way to learn how VBA works and how to reference and access elements is to record your macro then edit it in the VBA editor. This is how I learned VBA. It is based on visual basic so all the programming conventions of VB apply. Recording the macro lets you see how to access and do things.

you could use something like this:

var result = 0
Sheets("Sheet1").Select
result = Range("A1").Value * Range("B1").Value
Sheets("Sheet2").Select
Range("D1").Value = result

Alternatively you can also reference a cell using Cells(1,1).Value This way you can set variables and increment them as you wish. I think I am just not clear on exactly what you are trying to do but i hope this helps.

How to round up with excel VBA round()?

The answers here are kind of all over the map, and try to accomplish several different things. I'll just point you to the answer I recently gave that discusses the forced rounding UP -- i.e., no rounding toward zero at all. The answers in here cover different types of rounding, and ana's answer for example is for forced rounding up.

To be clear, the original question was how to "round normally" -- so, "for value > 0.5, round up. And for value < 0.5, round down".

The answer that I link to there discusses forced rounding up, which you sometimes also want to do. Whereas Excel's normal ROUND uses round-half-up, its ROUNDUP uses round-away-from-zero. So here are two functions that imitate ROUNDUP in VBA, the second of which only rounds to a whole number.

Function RoundUpVBA(InputDbl As Double, Digits As Integer) As Double

    If InputDbl >= O Then
        If InputDbl = Round(InputDbl, Digits) Then RoundUpVBA = InputDbl Else RoundUpVBA = Round(InputDbl + 0.5 / (10 ^ Digits), Digits)
    Else
        If InputDbl = Round(InputDbl, Digits) Then RoundUpVBA = InputDbl Else RoundUpVBA = Round(InputDbl - 0.5 / (10 ^ Digits), Digits)
    End If

End Function

Or:

Function RoundUpToWhole(InputDbl As Double) As Integer

    Dim TruncatedDbl As Double

    TruncatedDbl = Fix(InputDbl)

    If TruncatedDbl <> InputDbl Then
        If TruncatedDbl >= 0 Then RoundUpToWhole = TruncatedDbl + 1 Else RoundUpToWhole = TruncatedDbl - 1
    Else
        RoundUpToWhole = TruncatedDbl
    End If

End Function

Some of the answers above cover similar territory, but these here are self-contained. I also discuss in my other answer some one-liner quick-and-dirty ways to round up.

What is the Windows equivalent of the diff command?

Winmerge has a command line utility that might be worth checking out.

Also, you can use the graphical part of it too depending on what you need.

CakePHP select default value in SELECT input

FormHelper::select(string $fieldName, array $options, 
array $attributes)

$attributes['value'] to set which value should be selected default

<?php echo $this->Form->select('status', $list, array(
    'empty' => false, 
    'value' => 1)
); ?>

how to implement regions/code collapse in javascript

Blog entry here explains it and this MSDN question.

You have to use Visual Studio 2003/2005/2008 Macros.

Copy + Paste from Blog entry for fidelity sake:

  1. Open Macro Explorer
  2. Create a New Macro
  3. Name it OutlineRegions
  4. Click Edit macro and paste the following VB code:
Option Strict Off
Option Explicit Off

Imports System
Imports EnvDTE
Imports EnvDTE80
Imports System.Diagnostics
Imports System.Collections

Public Module JsMacros

    Sub OutlineRegions()
        Dim selection As EnvDTE.TextSelection = DTE.ActiveDocument.Selection

        Const REGION_START As String = "//#region"
        Const REGION_END As String = "//#endregion"

        selection.SelectAll()
        Dim text As String = selection.Text
        selection.StartOfDocument(True)

        Dim startIndex As Integer
        Dim endIndex As Integer
        Dim lastIndex As Integer = 0
        Dim startRegions As Stack = New Stack()

        Do
            startIndex = text.IndexOf(REGION_START, lastIndex)
            endIndex = text.IndexOf(REGION_END, lastIndex)

            If startIndex = -1 AndAlso endIndex = -1 Then
                Exit Do
            End If

            If startIndex <> -1 AndAlso startIndex < endIndex Then
                startRegions.Push(startIndex)
                lastIndex = startIndex + 1
            Else
                ' Outline region ...
                selection.MoveToLineAndOffset(CalcLineNumber(text, CInt(startRegions.Pop())), 1)
                selection.MoveToLineAndOffset(CalcLineNumber(text, endIndex) + 1, 1, True)
                selection.OutlineSection()

                lastIndex = endIndex + 1
            End If
        Loop

        selection.StartOfDocument()
    End Sub

    Private Function CalcLineNumber(ByVal text As String, ByVal index As Integer)
        Dim lineNumber As Integer = 1
        Dim i As Integer = 0

        While i < index
            If text.Chars(i) = vbCr Then
                lineNumber += 1
                i += 1
            End If

            i += 1
        End While

        Return lineNumber
    End Function

End Module
  1. Save the Macro and Close the Editor
  2. Now let's assign shortcut to the macro. Go to Tools->Options->Environment->Keyboard and search for your macro in "show commands containing" textbox
  3. now in textbox under the "Press shortcut keys" you can enter the desired shortcut. I use Ctrl+M+E. I don't know why - I just entered it first time and use it now :)

Removing the password from a VBA project

I found this here that describes how to set the VBA Project Password. You should be able to modify it to unset the VBA Project Password.

This one does not use SendKeys.

Let me know if this helps! JFV

How to add custom html attributes in JSX

uniqueId is custom attribute.

<a {...{ "uniqueId": `${item.File.UniqueId}` }}  href={item.File.ServerRelativeUrl} target='_blank'>{item.File.Name}</a>

ng-options with simple array init

If you setup your select like the following:

<select ng-model="myselect" ng-options="b for b in options track by b"></select>

you will get:

<option value="var1">var1</option>
<option value="var2">var2</option>
<option value="var3">var3</option>

working fiddle: http://jsfiddle.net/x8kCZ/15/

PHPMailer AddAddress()

Some great answers above, using that info here is what I did today to solve the same issue:

$to_array = explode(',', $to);
foreach($to_array as $address)
{
    $mail->addAddress($address, 'Web Enquiry');
}

'IF' in 'SELECT' statement - choose output value based on column values

SELECT id, amount
FROM report
WHERE type='P'

UNION

SELECT id, (amount * -1) AS amount
FROM report
WHERE type = 'N'

ORDER BY id;

Removing index column in pandas when reading a csv

DataFrames and Series always have an index. Although it displays alongside the column(s), it is not a column, which is why del df['index'] did not work.

If you want to replace the index with simple sequential numbers, use df.reset_index().

To get a sense for why the index is there and how it is used, see e.g. 10 minutes to Pandas.

Get value from JToken that may not exist (best practices)

Here is how you can check if the token exists:

if (jobject["Result"].SelectToken("Items") != null) { ... }

It checks if "Items" exists in "Result".

This is a NOT working example that causes exception:

if (jobject["Result"]["Items"] != null) { ... }

How do I automatically update a timestamp in PostgreSQL

Using 'now()' as default value automatically generates time-stamp.

Where to download Microsoft Visual c++ 2003 redistributable

Storm's answer is not correct. No hard feelings Storm, and apologies to the OP as I'm a bit late to the party here (wish I could have helped sooner, but I didn't run into the problem until today, or this stack overflow answer until I was figuring out a solution.)

The Visual C++ 2003 runtime was not available as a seperate download because it was included with the .NET 1.1 runtime.

If you install the .NET 1.1 runtime you will get msvcr71.dll installed, and in addition added to C:\WINDOWS\Microsoft.NET\Framework\v1.1.4322.

The .NET 1.1 runtime is available here: http://www.microsoft.com/downloads/en/details.aspx?familyid=262d25e3-f589-4842-8157-034d1e7cf3a3&displaylang=en (23.1 MB)

If you are looking for a file that ends with a "P" such as msvcp71.dll, this indicates that your file was compiled against a C++ runtime (as opposed to a C runtime), in some situations I noticed these files were only installed when I installed the full SDK. If you need one of these files, you may need to install the full .NET 1.1 SDK as well, which is available here: http://www.microsoft.com/downloads/en/details.aspx?FamilyID=9b3a2ca6-3647-4070-9f41-a333c6b9181d (106.2 MB)

After installing the SDK I now have both msvcr71.dll and msvcp71.dll in my System32 folder, and the application I'm trying to run (boomerang c++ decompiler) works fine without any missing DLL errors.

Also on a side note: be VERY aware of the difference between a Hotfix Update and a Regular Update. As noted in the linked KB932298 download (linked below by Storm): "Please be aware this Hotfix has not gone through full Microsoft product regression testing nor has it been tested in combination with other Hotfixes."

Hotfixes are NOT meant for general users, but rather users who are facing a very specific problem. As described in the article only install that Hotfix if you are have having specific daylight savings time issues with the rules that changed in 2007. -- Likely this was a pre-release for customers who "just couldn't wait" for the official update (probably for some business critical application) -- for regular users Windows Update should be all you need.

Thanks, and I hope this helps others who run into this issue!

Display number with leading zeros

df['Col1']=df['Col1'].apply(lambda x: '{0:0>5}'.format(x))

The 5 is the number of total digits.

I used this link: http://www.datasciencemadesimple.com/add-leading-preceding-zeros-python/

New lines (\r\n) are not working in email body

$mail = new PHPMailer;
$mail->isSMTP(); 
$mail->isHTML(true);

Insert this code after working all html tag like

<br> <p> in $mail->Body='Hello<br> how are you ?<b>';

How can I multiply and divide using only bit shifting and adding?

A procedure for dividing integers that uses shifts and adds can be derived in straightforward fashion from decimal longhand division as taught in elementary school. The selection of each quotient digit is simplified, as the digit is either 0 and 1: if the current remainder is greater than or equal to the divisor, the least significant bit of the partial quotient is 1.

Just as with decimal longhand division, the digits of the dividend are considered from most significant to least significant, one digit at a time. This is easily accomplished by a left shift in binary division. Also, quotient bits are gathered by left shifting the current quotient bits by one position, then appending the new quotient bit.

In a classical arrangement, these two left shifts are combined into left shifting of one register pair. The upper half holds the current remainder, the lower half initial holds the dividend. As the dividend bits are transferred to the remainder register by left shift, the unused least significant bits of the lower half are used to accumulate the quotient bits.

Below is x86 assembly language and C implementations of this algorithm. This particular variant of a shift & add division is sometimes referred to as the "no-performing" variant, as the subtraction of the divisor from the current remainder is not performed unless the remainder is greater than or equal to the divisor. In C, there is no notion of the carry flag used by the assembly version in the register pair left shift. Instead, it is emulated, based on the observation that the result of an addition modulo 2n can be smaller that either addend only if there was a carry out.

#include <stdio.h>
#include <stdlib.h>
#include <stdint.h>

#define USE_ASM 0

#if USE_ASM
uint32_t bitwise_division (uint32_t dividend, uint32_t divisor)
{
    uint32_t quot;
    __asm {
        mov  eax, [dividend];// quot = dividend
        mov  ecx, [divisor]; // divisor
        mov  edx, 32;        // bits_left
        mov  ebx, 0;         // rem
    $div_loop:
        add  eax, eax;       // (rem:quot) << 1
        adc  ebx, ebx;       //  ...
        cmp  ebx, ecx;       // rem >= divisor ?
        jb  $quot_bit_is_0;  // if (rem < divisor)
    $quot_bit_is_1:          // 
        sub  ebx, ecx;       // rem = rem - divisor
        add  eax, 1;         // quot++
    $quot_bit_is_0:
        dec  edx;            // bits_left--
        jnz  $div_loop;      // while (bits_left)
        mov  [quot], eax;    // quot
    }            
    return quot;
}
#else
uint32_t bitwise_division (uint32_t dividend, uint32_t divisor)
{
    uint32_t quot, rem, t;
    int bits_left = CHAR_BIT * sizeof (uint32_t);

    quot = dividend;
    rem = 0;
    do {
            // (rem:quot) << 1
            t = quot;
            quot = quot + quot;
            rem = rem + rem + (quot < t);

            if (rem >= divisor) {
                rem = rem - divisor;
                quot = quot + 1;
            }
            bits_left--;
    } while (bits_left);
    return quot;
}
#endif

How can I know which radio button is selected via jQuery?

In a JSF generated radio button (using <h:selectOneRadio> tag), you can do this:

radiobuttonvalue = jQuery("input[name='form_id\:radiobutton_id']:checked").val();

where selectOneRadio ID is radiobutton_id and form ID is form_id.

Be sure to use name instead id, as indicated, because jQuery uses this attribute (name is generated automatically by JSF resembling control ID).

What does the "+=" operator do in Java?

As others have said, it's bitwise XOR. If you want to raise a number to a given power, use Math.pow(a , b), where a is a number and b is the power.

Convert R vector to string vector of 1 element

Use the collapse argument to paste:

paste(a,collapse=" ")
[1] "aa bb cc"

List the queries running on SQL Server

There are various management views built into the product. On SQL 2000 you'd use sysprocesses. On SQL 2K5 there are more views like sys.dm_exec_connections, sys.dm_exec_sessions and sys.dm_exec_requests.

There are also procedures like sp_who that leverage these views. In 2K5 Management Studio you also get Activity Monitor.

And last but not least there are community contributed scripts like the Who Is Active by Adam Machanic.

How to find difference between two columns data?

IF the table is alias t

SELECT t.Present , t.previous, t.previous- t.Present AS Difference
FROM   temp1 as t

Force an SVN checkout command to overwrite current files

Try the --force option. svn help checkout gives the details.

In Java, how do you determine if a thread is running?

Have your thread notify some other thread when it’s finished. This way you’ll always know exactly what’s going on.

Specifing width of a flexbox flex item: width or basis?

The bottom statement is equivalent to:

.half {
   flex-grow: 0;
   flex-shrink: 0;
   flex-basis: 50%;
}

Which, in this case, would be equivalent as the box is not allowed to flex and therefore retains the initial width set by flex-basis.

Flex-basis defines the default size of an element before the remaining space is distributed so if the element were allowed to flex (grow/shrink) it may not be 50% of the width of the page.

I've found that I regularly return to https://css-tricks.com/snippets/css/a-guide-to-flexbox/ for help regarding flexbox :)

How do I kill all the processes in Mysql "show processlist"?

I recently needed to do this and I came up with this

-- GROUP_CONCAT turns all the rows into 1
-- @q:= stores all the kill commands to a variable
select @q:=GROUP_CONCAT(CONCAT('KILL ',ID) SEPARATOR ';')  
FROM information_schema.processlist 
-- If you don't need it, you can remove the WHERE command altogether
WHERE user = 'user';
-- Creates statement and execute it
PREPARE stmt FROM @q;
EXECUTE stmt;
DEALLOCATE PREPARE stmt;

That way, you don't need to store to file and run all queries with a single command.

Junit test case for database insert method with DAO and web service

/*

public class UserDAO {

public boolean insertUser(UserBean u) {
    boolean flag = false;
    MySqlConnection msq = new MySqlConnection();
    try {

        String sql = "insert into regis values(?,?,?,?,?)";

        Connection connection = msq.getConnection();
        PreparedStatement statement = null;
        statement = (PreparedStatement) connection.prepareStatement(sql);
        statement.setString(1, u.getname());
        statement.setString(2, u.getlname());
        statement.setString(3, u.getemail());
        statement.setString(4, u.getusername());
        statement.setString(5, u.getpasswords());
        statement.executeUpdate();

        flag = true;
    } catch (Exception e) {
    } finally {
        return flag;
    }

}

public String userValidate(UserBean u) {
    String login = "";
    MySqlConnection msq = new MySqlConnection();
    try {
        String email = u.getemail();
        String Pass = u.getpasswords();

        String sql = "SELECT name FROM regis WHERE email=? and passwords=?";
        com.mysql.jdbc.Connection connection = msq.getConnection();
        com.mysql.jdbc.PreparedStatement statement = null;
        ResultSet rs = null;
        statement = (com.mysql.jdbc.PreparedStatement) connection.prepareStatement(sql);
        statement.setString(1, email);
        statement.setString(2, Pass);
        rs = statement.executeQuery();
        if (rs.next()) {
            login = rs.getString("name");
        } else {
            login = "false";
        }

    } catch (Exception e) {
    } finally {
        return login;
    }
}

public boolean getmessage(UserBean u) {
    boolean flag = false;
    MySqlConnection msq = new MySqlConnection();
    try {


        String sql = "insert into feedback values(?,?)";

        Connection connection = msq.getConnection();
        PreparedStatement statement = null;
        statement = (PreparedStatement) connection.prepareStatement(sql);
        statement.setString(1, u.getemail());
        statement.setString(2, u.getfeedback());
        statement.executeUpdate();

        flag = true;
    } catch (Exception e) {
    } finally {
        return flag;
    }

}

public boolean insertOrder(cartbean u) {
    boolean flag = false;
    MySqlConnection msq = new MySqlConnection();
    try {

        String sql = "insert into cart (product_id, email, Tprice, quantity) values (?,?,2000,?)";

        Connection connection = msq.getConnection();
        PreparedStatement statement = null;
        statement = (PreparedStatement) connection.prepareStatement(sql);
        statement.setString(1, u.getpid());
        statement.setString(2, u.getemail());
        statement.setString(3, u.getquantity());

        statement.executeUpdate();

        flag = true;
    } catch (Exception e) {
        System.out.print("hi");
    } finally {
        return flag;
    }

}

}

dismissModalViewControllerAnimated deprecated

Now in iOS 6 and above, you can use:

[[Picker presentingViewController] dismissViewControllerAnimated:YES completion:nil];

Instead of:

[[Picker parentViewControl] dismissModalViewControllerAnimated:YES];

...And you can use:

[self presentViewController:picker animated:YES completion:nil];

Instead of

[self presentModalViewController:picker animated:YES];    

Subquery returned more than 1 value.This is not permitted when the subquery follows =,!=,<,<=,>,>= or when the subquery is used as an expression

Use In instead of =

 select * from dbo.books
 where isbn in (select isbn from dbo.lending 
                where act between @fdate and @tdate
                and stat ='close'
               )

or you can use Exists

SELECT t1.*,t2.*
FROM  books   t1 
WHERE  EXISTS ( SELECT * FROM dbo.lending t2 WHERE t1.isbn = t2.isbn and
                t2.act between @fdate and @tdate and t2.stat ='close' )

Angular 4 HttpClient Query Parameters

joshrathke is right.

In angular.io docs is written that URLSearchParams from @angular/http is deprecated. Instead you should use HttpParams from @angular/common/http. The code is quite similiar and identical to what joshrathke have written. For multiple parameters that are saved for instance in a object like

{
  firstParam: value1,
  secondParam, value2
}

you could also do

for(let property in objectStoresParams) {
  if(objectStoresParams.hasOwnProperty(property) {
    params = params.append(property, objectStoresParams[property]);
  }
}

If you need inherited properties then remove the hasOwnProperty accordingly.

How do I login and authenticate to Postgresql after a fresh install?

There are two methods you can use. Both require creating a user and a database.

By default psql connects to the database with the same name as the user. So there is a convention to make that the "user's database". And there is no reason to break that convention if your user only needs one database. We'll be using mydatabase as the example database name.

  1. Using createuser and createdb, we can be explicit about the database name,

    $ sudo -u postgres createuser -s $USER
    $ createdb mydatabase
    $ psql -d mydatabase
    

    You should probably be omitting that entirely and letting all the commands default to the user's name instead.

    $ sudo -u postgres createuser -s $USER
    $ createdb
    $ psql
    
  2. Using the SQL administration commands, and connecting with a password over TCP

    $ sudo -u postgres psql postgres
    

    And, then in the psql shell

    CREATE ROLE myuser LOGIN PASSWORD 'mypass';
    CREATE DATABASE mydatabase WITH OWNER = myuser;
    

    Then you can login,

    $ psql -h localhost -d mydatabase -U myuser -p <port>
    

    If you don't know the port, you can always get it by running the following, as the postgres user,

    SHOW port;
    

    Or,

    $ grep "port =" /etc/postgresql/*/main/postgresql.conf
    

Sidenote: the postgres user

I suggest NOT modifying the postgres user.

  1. It's normally locked from the OS. No one is supposed to "log in" to the operating system as postgres. You're supposed to have root to get to authenticate as postgres.
  2. It's normally not password protected and delegates to the host operating system. This is a good thing. This normally means in order to log in as postgres which is the PostgreSQL equivalent of SQL Server's SA, you have to have write-access to the underlying data files. And, that means that you could normally wreck havoc anyway.
  3. By keeping this disabled, you remove the risk of a brute force attack through a named super-user. Concealing and obscuring the name of the superuser has advantages.

Cheap way to search a large text file for a string

5000 lines isn't big (well, depends on how long the lines are...)

Anyway: assuming the string will be a word and will be seperated by whitespace...

lines=open(file_path,'r').readlines()
str_wanted="whatever_youre_looking_for"


    for i in range(len(lines)):
        l1=lines.split()
        for p in range(len(l1)):
            if l1[p]==str_wanted:
                #found
                # i is the file line, lines[i] is the full line, etc.

Using two CSS classes on one element

Remember that you can apply multiple classes to an element by separating each class with a space within its class attribute. For example:

<img class="class1 class2">

How can I extract a predetermined range of lines from a text file on Unix?

cat dump.txt | head -16224 | tail -258

should do the trick. The downside of this approach is that you need to do the arithmetic to determine the argument for tail and to account for whether you want the 'between' to include the ending line or not.

Is there an auto increment in sqlite?

As of today — June 2018


Here is what official SQLite documentation has to say on the subject (bold & italic are mine):

  1. The AUTOINCREMENT keyword imposes extra CPU, memory, disk space, and disk I/O overhead and should be avoided if not strictly needed. It is usually not needed.

  2. In SQLite, a column with type INTEGER PRIMARY KEY is an alias for the ROWID (except in WITHOUT ROWID tables) which is always a 64-bit signed integer.

  3. On an INSERT, if the ROWID or INTEGER PRIMARY KEY column is not explicitly given a value, then it will be filled automatically with an unused integer, usually one more than the largest ROWID currently in use. This is true regardless of whether or not the AUTOINCREMENT keyword is used.

  4. If the AUTOINCREMENT keyword appears after INTEGER PRIMARY KEY, that changes the automatic ROWID assignment algorithm to prevent the reuse of ROWIDs over the lifetime of the database. In other words, the purpose of AUTOINCREMENT is to prevent the reuse of ROWIDs from previously deleted rows.

An error has occured. Please see log file - eclipse juno

For me the problem was that I installed Java sdk 1.9 before installing eclipse. deleting it and installing Java sdk 1.8 instead fixed it. Also, if you are using mac, try

export JAVA_HOME=$(/usr/libexec/java_home)

and then

echo $JAVA_HOME

your should get something like

/Library/Java/JavaVirtualMachines/jdk1.8.0_144.jdk/Contents/Home

Update Top 1 record in table sql server

UPDATE TX_Master_PCBA
SET TIMESTAMP2 = '2013-12-12 15:40:31.593',
G_FIELD='0000'
WHERE TIMESTAMP2 IN 
(
   SELECT TOP 1 TIMESTAMP2
   FROM TX_Master_PCBA WHERE SERIAL_NO='0500030309'
   ORDER BY TIMESTAMP2 DESC   -- You need to decide what column you want to sort on
)

Counting DISTINCT over multiple columns

You can just use the Count Function Twice.

In this case, it would be:

SELECT COUNT (DISTINCT DocumentId), COUNT (DISTINCT DocumentSessionId) 
FROM DocumentOutputItems

How to show PIL Image in ipython notebook

much simpler in jupyter using pillow.

from PIL import Image
image0=Image.open('image.png')
image0

Determine if Android app is being used for the first time

Why not use the Database Helper ? This will have a nice onCreate which is only called the first time the app is started. This will help those people who want to track this after there initial app has been installed without tracking.

Visual Studio Code how to resolve merge conflicts with git?

For VS Code 1.38 or if you could not find any "lightbulb" button. Pay close attention to the greyed out text above the conflicts; there is a list of actions you can take.

Python class returning value

class MyClass():
    def __init__(self, a, b):
        self.value1 = a
        self.value2 = b

    def __call__(self):
        return [self.value1, self.value2]

Testing:

>>> x = MyClass('foo','bar')
>>> x()
['foo', 'bar']

Evaluate expression given as a string

Sorry but I don't understand why too many people even think a string was something that could be evaluated. You must change your mindset, really. Forget all connections between strings on one side and expressions, calls, evaluation on the other side.

The (possibly) only connection is via parse(text = ....) and all good R programmers should know that this is rarely an efficient or safe means to construct expressions (or calls). Rather learn more about substitute(), quote(), and possibly the power of using do.call(substitute, ......).

fortunes::fortune("answer is parse")
# If the answer is parse() you should usually rethink the question.
#    -- Thomas Lumley
#       R-help (February 2005)

Dec.2017: Ok, here is an example (in comments, there's no nice formatting):

q5 <- quote(5+5)
str(q5)
# language 5 + 5

e5 <- expression(5+5)
str(e5)
# expression(5 + 5)

and if you get more experienced you'll learn that q5 is a "call" whereas e5 is an "expression", and even that e5[[1]] is identical to q5:

identical(q5, e5[[1]])
# [1] TRUE

Paste text on Android Emulator

Using Visual Studio Emulator, Here's my method.

First Mound a virtual sd card:

  1. Use the Additional Tools (small >> icon) for the emulator and go to the SD Card tab.
  2. Select a folder on your computer to sync with the virtual SD card.
  3. Pull from SD card, which will create a folder structure on the selected folder.

Set up a text file to transfer text:

  1. Use Google Play Store to install a text editor of your choice
  2. Create a text file containing your text on you computer in the download directory of the virtual sd card directory you created before.

Whenever I need to send text to the clip board.

  1. Edit the text file created above.
  2. Go to Additional Tools (small >> icon) and chose Push To SD Card.
  3. Open the text file in the text editor I installed and copy the text to the clip board. (Hold down the mouse when the dialog opens, choose select all and then click the copy icon)

Once set up it pretty easy to repeat. The same method would be applicable to other emulators by you may need to use a different method to push your text file to emulator.

Does document.body.innerHTML = "" clear the web page?

As others were saying, an easy solution is to put your script at the bottom of the page, because all DOM elements are loaded synchronously from top to bottom. Otherwise, I think what you're looking for is document.onload(() => {callback_body}) or window.onload(() => {callback_body}) as Erik said. These allow you to execute you script when the dom:loaded event is fired as Douwe Maan said. Not sure about the properties of window.onload, but document.onload is triggered only after all elements, css, and scripts are loaded. In your case:

<script type="text/javascript">
    document.onload(() =>
        document.body.innerHTML = "";
    )
</script>

or, If you are using an old browser:

<script type="text/javascript">
    document.onload(function() {
        document.body.innerHTML = "";
    })
</script>

How do I compare two DateTime objects in PHP 5.2.8?

From the official documentation:

As of PHP 5.2.2, DateTime objects can be compared using comparison operators.

$date1 = new DateTime("now");
$date2 = new DateTime("tomorrow");

var_dump($date1 == $date2); // false
var_dump($date1 < $date2); // true
var_dump($date1 > $date2); // false

For PHP versions before 5.2.2 (actually for any version), you can use diff.

$datetime1 = new DateTime('2009-10-11'); // 11 October 2013
$datetime2 = new DateTime('2009-10-13'); // 13 October 2013

$interval = $datetime1->diff($datetime2);
echo $interval->format('%R%a days'); // +2 days

Is it possible that one domain name has multiple corresponding IP addresses?

This is round robin DNS. This is a quite simple solution for load balancing. Usually DNS servers rotate/shuffle the DNS records for each incoming DNS request. Unfortunately it's not a real solution for fail-over. If one of the servers fail, some visitors will still be directed to this failed server.

'profile name is not valid' error when executing the sp_send_dbmail command

You need to grant the user or group rights to use the profile. They need to be added to the msdb database and then you will see them available in the mail wizard when you are maintaining security for mail.

Read up the security here: http://msdn.microsoft.com/en-us/library/ms175887.aspx

See a listing of mail procedures here: http://msdn.microsoft.com/en-us/library/ms177580.aspx

Example script for 'TestUser' to use the profile named 'General Admin Mail'.


USE [msdb]
GO
CREATE USER [TestUser] FOR LOGIN [testuser]
GO
USE [msdb]
GO
EXEC sp_addrolemember N'DatabaseMailUserRole', N'TestUser'
GO

EXECUTE msdb.dbo.sysmail_add_principalprofile_sp
    @profile_name = 'General Admin Mail',
    @principal_name = 'TestUser',
    @is_default = 1 ;

Detecting a mobile browser

//true / false
function isMobile()
{
   return (/Android|webOS|iPhone|iPad|iPod|BlackBerry/i.test(navigator.userAgent) ); 
}

also you can follow this tutorial to detect a specific mobile. Click here.

how to execute php code within javascript

You could use http://phpjs.org/ http://locutus.io/php/ it ports a bunch of PHP functionality to javascript, but if it's just echos, and the script is in a php file, you could do something like this:

alert("<?php echo "asdasda";?>");

don't worry about the shifty-looking use of double-quotes, PHP will render that before the browser sees it.

as for using ajax, the easiest way is to use a library, like jQuery. With that you can do:

$.ajax({
  url: 'test.php',
  success: function(data) {
    $('.result').html(data);
  }
});

and test.php would be:

<?php 
  echo 'asdasda';
?>

it would write the contents of test.php to whatever element has the result class.

Typing Greek letters etc. in Python plots

If you want tho have a normal string infront of the greek letter make sure that you have the right order:

plt.ylabel(r'Microstrain [$\mu \epsilon$]')

How to remove the last character from a string?

Since we're on a subject, one can use regular expressions too

"aaabcd".replaceFirst(".$",""); //=> aaabc  

How to find out if a file exists in C# / .NET?

System.IO.File:

using System.IO;

if (File.Exists(path)) 
{
    Console.WriteLine("file exists");
} 

Simultaneously merge multiple data.frames in a list

I will reuse the data example from @PaulRougieux

x <- data_frame(i = c("a","b","c"), j = 1:3)
y <- data_frame(i = c("b","c","d"), k = 4:6)
z <- data_frame(i = c("c","d","a"), l = 7:9)

Here's a short and sweet solution using purrr and tidyr

library(tidyverse)

 list(x, y, z) %>% 
  map_df(gather, key=key, value=value, -i) %>% 
  spread(key, value)

How to edit my Excel dropdown list?

Attribute_Brands is a named range.

On any worksheet (tab) press F5 and type Attribute_Brands into the reference box and click on the OK button.

This will take you to the named range.

The data in it can be updated by typing new values into the cells.

The named range can be altered via the 'Insert - Name - Define' menu.

PHP, get file name without file extension

The existing solutions fail when there are multiple parts to an extension. The function below works for multiple parts, a full path or just a a filename:

function removeExt($path)
{
    $basename = basename($path);
    return strpos($basename, '.') === false ? $path : substr($path, 0, - strlen($basename) + strlen(explode('.', $basename)[0]));
}

echo removeExt('https://example.com/file.php');
// https://example.com/file
echo removeExt('https://example.com/file.tar.gz');
// https://example.com/file
echo removeExt('file.tar.gz');
// file
echo removeExt('file');
// file

What is the correct JSON content type?

If you are using Ubuntu or Debian and you serve .json files through Apache, you might want to serve the files with the correct content type. I am doing this primarily because I want to use the Firefox extension JSONView

The Apache module mod_mime will help to do this easily. However, with Ubuntu you need to edit the file /etc/mime.types and add the line

application/json json

Then restart Apache:

sudo service apache2 restart

jQuery: what is the best way to restrict "number"-only input for textboxes? (allow decimal points)

This is very simple that we have already a javascript inbuilt function "isNaN" is there.

$("#numeric").keydown(function(e){
  if (isNaN(String.fromCharCode(e.which))){ 
    return false; 
  }
});

Java: Reading integers from a file into an array

It looks like Java is trying to convert an empty string into a number. Do you have an empty line at the end of the series of numbers?

You could probably fix the code like this

String s = in.readLine();
int i = 0;

while (s != null) {
    // Skip empty lines.
    s = s.trim();
    if (s.length() == 0) {
        continue;
    }

    tall[i] = Integer.parseInt(s); // This is line 19.
    System.out.println(tall[i]);
    s = in.readLine();
    i++;
}

in.close();

Replace text inside td using jQuery having td containing other elements

Remove the textnode, and replace the <b> tag with whatever you need without ever touching the inputs :

$('#demoTable').find('tr > td').contents().filter(function() {
    return this.nodeType===3;
}).remove().end().end()
  .find('b').replaceWith($('<span />', {text: 'Hello Kitty'}));

FIDDLE

PHP Multiple Checkbox Array

if (isset($_POST['submit'])) {

for($i = 0; $i<= 3; $i++){

    if(isset($_POST['books'][$i]))

        $book .= ' '.$_POST['books'][$i];
}

Show all tables inside a MySQL database using PHP?

SHOW TABLE_NAME is not valid. Try SHOW TABLES

TD

Postman: sending nested JSON object

Select the body tab and select application/json in the Content-Type drop-down and add a body like this:

{
  "Username":"ABC",
  "Password":"ABC"
}

enter image description here

conflicting types error when compiling c program using gcc

You have to declare your functions before main()

(or declare the function prototypes before main())

As it is, the compiler sees my_print (my_string); in main() as a function declaration.

Move your functions above main() in the file, or put:

void my_print (char *);
void my_print2 (char *);

Above main() in the file.

How do I POST a x-www-form-urlencoded request using Fetch?

Just set the body as the following

var reqBody = "username="+username+"&password="+password+"&grant_type=password";

then

fetch('url', {
      method: 'POST',
      headers: {
          //'Authorization': 'Bearer token',
          'Content-Type': 'application/x-www-form-urlencoded; charset=UTF-8'
      },
      body: reqBody
  }).then((response) => response.json())
      .then((responseData) => {
          console.log(JSON.stringify(responseData));
      }).catch(err=>{console.log(err)})

Is there a Java equivalent or methodology for the typedef keyword in C++?

You could use an Enum, although that's semantically a bit different than a typedef in that it only allows a restricted set of values. Another possible solution is a named wrapper class, e.g.

public class Apple {
      public Apple(Integer i){this.i=i; }
}

but that seems way more clunky, especially given that it's not clear from the code that the class has no other function than as an alias.

Convert Numeric value to Varchar

i think it should be

select convert(varchar(10),StandardCost) +'S' from DimProduct where ProductKey = 212

or

select cast(StandardCost as varchar(10)) + 'S' from DimProduct where ProductKey = 212

How to reload .bashrc settings without logging out and back in again?

exec bash is a great way to re-execute and launch a new shell to replace current. just to add to the answer, $SHELL returns the current shell which is bash. By using the following, it will reload the current shell, and not only to bash.

exec $SHELL -l;

ngrok command not found

Ngrok can be installed with Yarn , then you can run by power Sheel. it's was the only way that worked for me in windows 10 . In the begin you need to install the Node : https://nodejs.org/en/. and the yarn: https://nodejs.org/en/.

My Routes are Returning a 404, How can I Fix Them?

Just Run in your terminal.

 composer dump-autoload

How to change RGB color to HSV?

Note that Color.GetSaturation() and Color.GetBrightness() return HSL values, not HSV.
The following code demonstrates the difference.

Color original = Color.FromArgb(50, 120, 200);
// original = {Name=ff3278c8, ARGB=(255, 50, 120, 200)}

double hue;
double saturation;
double value;
ColorToHSV(original, out hue, out saturation, out value);
// hue        = 212.0
// saturation = 0.75
// value      = 0.78431372549019607

Color copy = ColorFromHSV(hue, saturation, value);
// copy = {Name=ff3278c8, ARGB=(255, 50, 120, 200)}

// Compare that to the HSL values that the .NET framework provides: 
original.GetHue();        // 212.0
original.GetSaturation(); // 0.6
original.GetBrightness(); // 0.490196079

The following C# code is what you want. It converts between RGB and HSV using the algorithms described on Wikipedia. The ranges are 0 - 360 for hue, and 0 - 1 for saturation or value.

public static void ColorToHSV(Color color, out double hue, out double saturation, out double value)
{
    int max = Math.Max(color.R, Math.Max(color.G, color.B));
    int min = Math.Min(color.R, Math.Min(color.G, color.B));

    hue = color.GetHue();
    saturation = (max == 0) ? 0 : 1d - (1d * min / max);
    value = max / 255d;
}

public static Color ColorFromHSV(double hue, double saturation, double value)
{
    int hi = Convert.ToInt32(Math.Floor(hue / 60)) % 6;
    double f = hue / 60 - Math.Floor(hue / 60);

    value = value * 255;
    int v = Convert.ToInt32(value);
    int p = Convert.ToInt32(value * (1 - saturation));
    int q = Convert.ToInt32(value * (1 - f * saturation));
    int t = Convert.ToInt32(value * (1 - (1 - f) * saturation));

    if (hi == 0)
        return Color.FromArgb(255, v, t, p);
    else if (hi == 1)
        return Color.FromArgb(255, q, v, p);
    else if (hi == 2)
        return Color.FromArgb(255, p, v, t);
    else if (hi == 3)
        return Color.FromArgb(255, p, q, v);
    else if (hi == 4)
        return Color.FromArgb(255, t, p, v);
    else
        return Color.FromArgb(255, v, p, q);
}

Why catch and rethrow an exception in C#?

My main reason for having code like:

try
{
    //Some code
}
catch (Exception e)
{
    throw;
}

is so I can have a breakpoint in the catch, that has an instantiated exception object. I do this a lot while developing/debugging. Of course, the compiler gives me a warning on all the unused e's, and ideally they should be removed before a release build.

They are nice during debugging though.

How can I remove the "No file chosen" tooltip from a file input in Chrome?

This is a tricky one. I could not find a way to select the 'no file chosen' element so I created a mask using the :after pseudo selector.

My solution also requires the use of the following pseudo selector to style the button:

::-webkit-file-upload-button

Try this: http://jsfiddle.net/J8Wfx/1/

FYI: This will only work in webkit browsers.

P.S if anyone knows how to view webkit pseudo selectors like the one above in the webkit inspector please let me know

Split a large pandas dataframe

you can use list comprehensions to do this in a single line

n = 4
chunks = [df[i:i+n] for i in range(0,df.shape[0],n)]

Using OpenSSL what does "unable to write 'random state'" mean?

You should set the $RANDFILE environment variable and/or create $HOME/.rnd file. (OpenSSL FAQ). (Of course, you should have rights to that file. Others answers here are about that. But first you should have the file and a reference to it.)

Up to version 0.9.6 OpenSSL wrote the seeding file in the current directory in the file ".rnd". At version 0.9.6a you have no default seeding file. OpenSSL 0.9.6b and later will behave similarly to 0.9.6a, but will use a default of "C:\" for HOME on Windows systems if the environment variable has not been set.

If the default seeding file does not exist or is too short, the "PRNG not seeded" error message may occur.

The $RANDFILE environment variable and $HOME/.rnd are only used by the OpenSSL command line tools. Applications using the OpenSSL library provide their own configuration options to specify the entropy source, please check out the documentation coming the with application.

How do I minimize the command prompt from my bat file

One option is to find one of the various utilities that can change the window state of the currently running console window and make a call to it from within the batch script.

You can run it as the first thing in your batch script. Here are two such tools:

min.exe http://www.paulsadowski.com/wsh/cmdprogs.htm

cmdow http://www.commandline.co.uk/cmdow/index.html

HTML Input Type Date, Open Calendar by default

This is not possible with native HTML input elements. You can use webshim polyfill, which gives you this option by using this markup.

<input type="date" data-date-inline-picker="true" />

Here is a small demo

dropping a global temporary table

Step 1. Figure out which errors you want to trap:

If the table does not exist:

SQL> drop table x;
drop table x
           *
ERROR at line 1:
ORA-00942: table or view does not exist

If the table is in use:

SQL> create global temporary table t (data varchar2(4000));

Table created.

Use the table in another session. (Notice no commit or anything after the insert.)

SQL> insert into t values ('whatever');

1 row created.

Back in the first session, attempt to drop:

SQL> drop table t;
drop table t
           *
ERROR at line 1:
ORA-14452: attempt to create, alter or drop an index on temporary table already in use

So the two errors to trap:

  1. ORA-00942: table or view does not exist
  2. ORA-14452: attempt to create, alter or drop an index on temporary table already in use

See if the errors are predefined. They aren't. So they need to be defined like so:

create or replace procedure p as
    table_or_view_not_exist exception;
    pragma exception_init(table_or_view_not_exist, -942);
    attempted_ddl_on_in_use_GTT exception;
    pragma exception_init(attempted_ddl_on_in_use_GTT, -14452);
begin
    execute immediate 'drop table t';

    exception 
        when table_or_view_not_exist then
            dbms_output.put_line('Table t did not exist at time of drop. Continuing....');

        when attempted_ddl_on_in_use_GTT then
            dbms_output.put_line('Help!!!! Someone is keeping from doing my job!');
            dbms_output.put_line('Please rescue me');
            raise;
end p;

And results, first without t:

SQL> drop table t;

Table dropped.

SQL> exec p;
Table t did not exist at time of drop. Continuing....

PL/SQL procedure successfully completed.

And now, with t in use:

SQL> create global temporary table t (data varchar2(4000));

Table created.

In another session:

SQL> insert into t values (null);

1 row created.

And then in the first session:

SQL> exec p;
Help!!!! Someone is keeping from doing my job!
Please rescue me
BEGIN p; END;

*
ERROR at line 1:
ORA-14452: attempt to create, alter or drop an index on temporary table already in use
ORA-06512: at "SCHEMA_NAME.P", line 16
ORA-06512: at line 1

SQL Server 2005 Using CHARINDEX() To split a string

Create FUNCTION [dbo].[fnSplitString] 
( 
    @string NVARCHAR(200), 
    @delimiter CHAR(1) 
) 
RETURNS @output TABLE(splitdata NVARCHAR(10) 
) 
BEGIN 
    DECLARE @start INT, @end INT 
    SELECT @start = 1, @end = CHARINDEX(@delimiter, @string) 
    WHILE @start < LEN(@string) + 1 BEGIN 
        IF @end = 0  
            SET @end = LEN(@string) + 1

        INSERT INTO @output (splitdata)  
        VALUES(SUBSTRING(@string, @start, @end - @start)) 
        SET @start = @end + 1 
        SET @end = CHARINDEX(@delimiter, @string, @start)

    END 
    RETURN 

END**strong text**

How can I get argv[] as int?

/*

    Input from command line using atoi, and strtol 
*/

#include <stdio.h>//printf, scanf
#include <stdlib.h>//atoi, strtol 

//strtol - converts a string to a long int 
//atoi - converts string to an int 

int main(int argc, char *argv[]){

    char *p;//used in strtol 
    int i;//used in for loop

    long int longN = strtol( argv[1],&p, 10);
    printf("longN = %ld\n",longN);

    //cast (int) to strtol
    int N = (int) strtol( argv[1],&p, 10);
    printf("N = %d\n",N);

    int atoiN;
    for(i = 0; i < argc; i++)
    {
        //set atoiN equal to the users number in the command line 
        //The C library function int atoi(const char *str) converts the string argument str to an integer (type int).
        atoiN = atoi(argv[i]);
    }

    printf("atoiN = %d\n",atoiN);
    //-----------------------------------------------------//
    //Get string input from command line 
    char * charN;

    for(i = 0; i < argc; i++)
    {
        charN = argv[i];
    }

    printf("charN = %s\n", charN); 

}

Hope this helps. Good luck!

How to use dashes in HTML-5 data-* attributes in ASP.NET MVC

This problem has been addressed in ASP.Net MVC 3. They now automatically convert underscores in html attribute properties to dashes. They got lucky on this one, as underscores are not legal in html attributes, so MVC can confidently imply that you'd like a dash when you use an underscore.

For example:

@Html.TextBoxFor(vm => vm.City, new { data_bind = "foo" })

will render this in MVC 3:

<input data-bind="foo" id="City" name="City" type="text" value="" />

If you're still using an older version of MVC, you can mimic what MVC 3 is doing by creating this static method that I borrowed from MVC3's source code:

public class Foo {
    public static RouteValueDictionary AnonymousObjectToHtmlAttributes(object htmlAttributes) {
        RouteValueDictionary result = new RouteValueDictionary();
        if (htmlAttributes != null) {
            foreach (System.ComponentModel.PropertyDescriptor property in System.ComponentModel.TypeDescriptor.GetProperties(htmlAttributes)) {
                result.Add(property.Name.Replace('_', '-'), property.GetValue(htmlAttributes));
            }
        }
        return result;
    }
}

And then you can use it like this:

<%: Html.TextBoxFor(vm => vm.City, Foo.AnonymousObjectToHtmlAttributes(new { data_bind = "foo" })) %>

and this will render the correct data-* attribute:

<input data-bind="foo" id="City" name="City" type="text" value="" />

How to fill a Javascript object literal with many static key/value pairs efficiently?

Give this a try:

var map = {"aaa": "rrr", "bbb": "ppp"};

How to wait in a batch script?

Well, does sleep even exist on your Windows XP box? According to this post: http://malektips.com/xp_dos_0002.html sleep isn't available on Windows XP, and you have to download the Windows 2003 Resource Kit in order to get it.

Chakrit's answer gives you another way to pause, too.

Try running sleep 10 from a command prompt.

How to check Grants Permissions at Run-Time?

Try this instead simple request code
https://www.learn2crack.com/2015/10/android-marshmallow-permissions.html

public static final int REQUEST_ID_MULTIPLE_PERMISSIONS = 1;

private  boolean checkAndRequestPermissions() {
    int camera = ContextCompat.checkSelfPermission(this, android.Manifest.permission.CAMERA);
    int storage = ContextCompat.checkSelfPermission(this, android.Manifest.permission.WRITE_EXTERNAL_STORAGE);
    int loc = ContextCompat.checkSelfPermission(this, android.Manifest.permission.ACCESS_COARSE_LOCATION);
    int loc2 = ContextCompat.checkSelfPermission(this, android.Manifest.permission.ACCESS_FINE_LOCATION);
    List<String> listPermissionsNeeded = new ArrayList<>();

    if (camera != PackageManager.PERMISSION_GRANTED) {
        listPermissionsNeeded.add(android.Manifest.permission.CAMERA);
    }
    if (storage != PackageManager.PERMISSION_GRANTED) {
        listPermissionsNeeded.add(android.Manifest.permission.WRITE_EXTERNAL_STORAGE);
    }
    if (loc2 != PackageManager.PERMISSION_GRANTED) {
        listPermissionsNeeded.add(android.Manifest.permission.ACCESS_FINE_LOCATION);
    }
    if (loc != PackageManager.PERMISSION_GRANTED) {
        listPermissionsNeeded.add(android.Manifest.permission.ACCESS_COARSE_LOCATION);
    }
    if (!listPermissionsNeeded.isEmpty())
    {
        ActivityCompat.requestPermissions(this,listPermissionsNeeded.toArray
                (new String[listPermissionsNeeded.size()]),REQUEST_ID_MULTIPLE_PERMISSIONS);
        return false;
    }
    return true;
}